Aku 2
Aku 2
Aku 2
SYSTEM:
1. Pulmonary venolabar syndrome
2. bronchogenic cyst
3. invasive aspergillosis in transplant patient
4. pulmonary sarcoidosis
5. complications of thorocotomy
6. cystic fibrosis
7. Thymoma
8. Empyema
9. lung metastasis
10. Transient tachypnea of the newborn
11. congenital lobar emphysema
1. A 21 year gentleman has a pre employment chest x-ray with no known co-morbid. His x-ray chest
shows a linear opacity at right base projecting below the diaphragm with loss of volume in the right
hemithorax.
The most consistent feature of this condition is:
a) Absence or interruption of IVC
b) Arterial supply wholly from thoracic aorta
c) Hypogenetic lung
d) Pulmonary sequestration
e) Systemic arterializations of lung
Reference: Dahnert 6th Edition page 486 (answer: Hypogenetic lung, C2, easy, essential)
2. An asymptomatic 49 year old gentleman on routine chest examination had a large round mass in the
right cadiophrenic angle, subsequent CT scan for further assessment revealed an oval mass of
predominantly fat density separate from the heart below the tracheal bifurcation: what is the most likely
diagnosis?
a) Bochdalek’s hernia
b) Lymphangioma
c) Morgagni’s hernia
d) Pleuropericardial cyst
e) Thymoma
David Sutton 7th edition: 67 (answer: Morgagni’s hernia, C2, easy essential)
3. A 25 years gentleman presents with stridor and dysphagia, he undergoes a CT examination
which shows a unilocular cysts with water contents in the mediastinum.
The most common location for this cyst is:
a. Esophageal wall
b. Left upper lobe
c. Paratracheal
d. Pericarinal
e. Retrocardiac
Reference: Dahnert 6th Edition page 478 (answer: Pericarinal,C1, easy, essential)
4. A 39 year lady developed cough after renal transplant on 10th post-operative day. Her
physician ordered a HRCT examination of chest. Her CT showed a nodule with a halo of
increased attenuation in left upper lobe.
Patient has most likely acquired:
a) Candidiasis
b) cytomegalovirus
c) Histoplasmosis
d) Invasive aspergillosis
e) Pneumocyctis carinii
Reference: Sutton 7th Edition, Chapter 5, page 148 (answer: Invasive aspergillosis, C2, moderate,
essential)
5. A 30 year gentleman underwent plain x-ray chest as a pre-requisite for employment in AKU.
His x-ray shows bilateral hilar and right para-tracheal lymphadenopathy. Further lab
investigations sow normal ESR and LDH. His clinician suspects sarcoidosis.
The most likely stage of disease is:
a) 0
b) I
c) II
d) III
e) IV
Reference: 6th edition Dahnert page 538, chapter 7 of Sutton (answer: stage I, C1, easy,
essential)
6. A 16 year boy underwent pneumonectomy of left lung due to end stage bronchiectasis. Three
days post operatively he suddenly develops an air-fluid level in left hemithorax
The most likely cause of this finding is:
a) Broncho-pleural fistula
b) Broncho-pulmonary fistula
c) Diaphragmatic hernia
d) Empyema
e) Lung abscess
Reference: seventh edition Sutton page 225-6, chapter 8 (answer: Broncho-pleural fistula, C2,
moderate, important)
7. A 6 year old boy presented with cough and respiratory distress for three days. He suffers from
repeated chest infections since birth. His lab investigations show decreased PABA excretion.
The most pathagnomic finding on CXR is:
a) Cystic bronchiectasis
b) Glove finger appearance
c) Hilar lymphadenopathy
d) Lobar atelactasis
e) Peri-hilar linear densities
Reference: 6th edition Dahnert page 488, chapter 9 of Sutton (answer: Cystic bronchiectasis,
C2, moderate, essential)
12. A three months old baby boy undergoes x-ray chest for repeated respiratory tract infections.
His x-ray shows gross over inflation of his right lung which is hypovascular, resulting in
mediastinal shift towards left side.
The most characteristic feature of this condition is:
a) Associated with polycystic kidney disease
b) Is associated with decrease alveoli number
c) Progressive alveolar wall destruction
d) Progressive over distension of the lobe
e) The lower lobes are frequently involved
Reference: Sutton 7th edition, page: 251-2 (answer: Progressive over distension of the lobe,
C3, easy, essential)
13. A 60 year old gentle man smoker for 35 years is diagnosed as case of lung carcinoma on bronchial
brushings. On CT chest collapse of right lower lobe is reported. For further evaluation MRI chest is
planned by mutual consensus of radiologist and physician.
To distinguish between collapse lungs from tumor the most important MR sequence is:
a) ADC mapping
b) DW sequence
c) FLAIR sequence
d) T1W sequence
e) T2 sequence
David Sutton 7th edition page 125 (answer: T2 sequence, C1, easy, essential)
14. A 60 year old gentleman is diagnosed as a case of lung carcinoma presents with severe chest wall pain
on right side. His treating physician suspects that the tumor has infiltrated the chest wall.
The most helpful examination to determine earliest chest wall invasion in this case is:
a) CT
b) Fluoroscopy
c) MRI
d) Ultrasound
e) X-ray chest
David Sutton 7th edition page 125(answer: MRI, C1, easy, essential)
15. A 60 year old gentleman who is a smoker for last 40 years is diagnosed as a case of lung carcinoma
presents with severe ischemia of right hand with excruciating pain for 24 hours. His treating physician
suspects that the tumour has infiltrated the vessel.
The best modality to determine earliest vascular invasion is:
a. CT
b. Fluoroscopy
c. MRI
d. Ultrasound
e. X-ray chest
David Sutton 7th edition page 126 (answer: MRI, C1, easy, essential)
16. A 50 year old white gentleman complains of persistent cough, wheezing, hemoptysis, flushing, diarrhea
and vomiting for the last 3 years. On laboratory investigations his urinary 5-HIAA levels are very high.
His chest X- ray he has recurrent unifocal pneumonitis. His CT chest reveals mass lesion.
The most common location of this mass lesion is:
a. Bronchial wall
b. Central Lobar
c. Endobronchial
d. exobronchial
e. Submucosal
David Sutton 7th edition page 122 (answer: central lobar, C1, moderate, important)
17. A 50 years white gentleman complains of persistent cough, wheezing, hemoptysis, flushing, diarrhea
and vomiting for the last 3 years. On chest X- ray he has recurrent unifocal pneumonitis. On laboratory
investigations his urinary 5-HIAA levels are very high. His CT chest is performed which reveals mass
lesion in main segmental bronchus.
The diagnostic feature of this mass lesion is:
a. Calcification on CT
b. Cystic spaces
c. No enhacement on CT
d. Non circumscribed mass
e. Pulmonary embolism
David Sutton 7th edition page 123 (answer: calcification on CT, C1, easy, important)
18. 65 year old gentleman presented with single episode of hemoptysis, and with vague chest pain and
cough since last 6 months has. Past history is insignificant.On CT chest there is a lobulated smooth
mass lesion with pop corn calcifications and fat density.
The most likely diagnosis is:
a. Amyloidoma
b. Granulomatous disease
c. Lipid pneumonia
d. Metastatic mucinous adenocarcinoma
e. Wegners granulomatosis
19. 65 year old gentleman presented with single episode of hemoptysis, and with vague chest pain and
cough since last 6 months has. Past history is insignificant.On CT chest there is a lobulated smooth
mass lesion with pop corn calcifications and fat density.
The common location of this lesion is:
a. Central
b. Chest wall
c. CP angle
d. Endobronchial
e. Peripheral
Dahnert 6th edition page 497
20. A 30 year old gentleman has painless lymphadenopathy, unexplained fever, night sweats and weight
loss. CT guided biopsy of the mediastinal lymph nodes revealed Reed-Sternberg cells. The frequency
of involvement of lung prachyma by this disease is:
a. 10%
b. 20%
c. 30%
d. 40%
e. 50%
David Sutton 7th edition page 118
21. A 30years gentle man has painless lymphadenopathy, unexplained fever, night sweats and weight loss.
CT guided biopsy of the mediastinal lymph nodes revealed Reed-Sternberg cells.
The pleural effusion in this patient is most likely because of:
a. Inflammatory response of pleura
b. Involvement of pleura
c. Loss of renal functions
d. Lymphatic obstruction
e. Protein loss / hypoproteinemia
David Sutton 7th edition page 118
22. A 30 year old gentleman has painless lymphadenopathy, unexplained fever, night sweats and weight
loss. CT guided biopsy of the mediastinal lymph nodes revealed Reed-Sternberg cells.
Involvement of bronchial wall by this disease process may lead to:
a. Collapse/Consolidation
b. Pleural effusion
c. Pulmonary arterial hypertension
d. COPD
e. Cystic spaces in lung parenchyma
David Sutton 7th edition page 118
23. A 30 year old gentleman has painless lymphadenopathy, unexplained fever, night sweats and weight
loss. CT guided biopsy of the mediastinal lymph nodes revealed Reed-Sternberg cells.
The most common extra nodal site of this disease involvement is:
a. Bone
b. Chest wall
c. CNS
d. Head and neck
e. Thymus
Dahnert 6th edition, page 500
24. 60 year old lady smoker for 35 years presents with productive cough and weight loss for 6 months. Her
CXR shows an ill defined right hilar mass. She undergoes CT chest for further evaluation and staging
of the mass.
The sign which represent non- operability is:
a. Tumor invading the visceral pleura
b. Tumor involve main bronchus >2cm distal to carina
c. Tumor involving carina
d. Tumor size is less than 2 cm
e. Tumor with pleural effusion
David Sutton 7th edition page 117
25. 60 year old lady smoker for 35 years presents with productive cough and weight loss for 6 months. She
undergoes CT chest which reveals a cavitating lesion on CT scan.
The most likely type of lung cancer in this would be:
a) Adeno carcinomas
b) Mixed cell carcinomas
c) Oat cell carcinomas
d) Small cell carcinomas
e) Squamous cell cancer
David Sutton 7th edition page 108
26. A 50 year old gentleman smoker and worker of nickel industry complain of fever and weight loss. He
has a patch of consolidation in left lower lobe on CXR. After 7 days of antibiotics follow up X-ray did
not reveal any improvement. He was admitted and IV antibiotics were given for another week which
also did not reveal any improvement.
The next step would be:
a. CT guided biopsy
b. CT scan chest with contrast
c. Follow up X-ray after 1 week without treatment.
d. Prescribe antibiotics for another week
e. Ultrasound chest
David Sutton 7th edition page 108
27. A 50 year gentle man, smoker and worker of nickel industry complain of fever and weight loss. On
examination he has ptosis of right eye with miosis, along with anhydrosis of right side of face.
The most likely finding on CXR will be:
a. Collapse of right lung
b. Erosion of 1st rib on right side
c. Hilar lymph node on right side
d. Pleural effusion on right side
e. Pneumothorax on right side
David Sutton 7th edition page 110
28. A 60 year old male worker of asbestos industry presented with finger clubbing and joint pain and
hypoglycemia. The chest X-ray showed well defined lobulated mass adjacent to chest wall. The latent
period of asbestos exposure and development of this condition is:
a. 1-5 years
b. 5-10 years
c. 10-20 years
d. 20-40 years
e. 40-60 years
David Sutton 7th edition page 102
29. A 60 year old male worker of asbestos industry presented with finger clubbing and joint pain and
hypoglycemia. The chest X-ray showed well defined lobulated mass adjacent to chest wall. CT guided
biopsy of this lesion was done which shows that this disease is malignant.
The occasional complication of this biopsy would be:
a. Development of ipsilateral pleural effusion
b. Massive bleeding at puncture site
c. Seedling of tumor along biopsy tract
d. Severe pain at puncture site
e. Tension Pneumothorax with mediastinal shift
David Sutton 7th edition page 103
30. A 40 years old lady underwent laprotomy for small bowel obstruction. On 7th post operative day she
developed high grade fever with chills and right upper quadrant abdominal pain. She is suspected to
have developed subphrenic abscess.
The plain film finding would not include:
a. Abnormal gas shadow between right hemidiaphragm
b. Elevated right hemidiaphragm
c. Erosion of right sided lower ribs
d. Right sided basal atelactasis
e. Right sided pleural effusion
David Sutton 7th edition page 53
31. A 46 year old man presented with pain in right arm and right side of the neck, his x-ray cervical spine
revealed dense pleural shadowing at the right lung apex and eroded pedicle of first thoracic vertebrae,
his symptoms are due to tumor involving:
a. Axillary artery
b. Brachial plexus
c. Stellate ganglion
d. Subclavian artery
e. Sympathetic chain
Dahnert Radiology Review Manual 6th edition: 476 and David Sutton 7th edition: 109
34. A young man developed sudden hypoxia requiring high ventilation pressures in recovery room
immediately after aortic valve replacement surgery under general anesthesia. His portable chest x-ray
showed massive pulmonary edema. His signs and symptoms are pathognomonic of
a. Complication of halothane
b. Dressler’s syndrome
c. Mendelson’s syndrome
d. Myocardial infarction
e. Pulmonary embolism
David Sutton 7th edition: 139
35. A 50 year old smoker underwent HRCT examination which revealed emphysematous spaces more than
1 centimeter in diameter with central dot/line and without definable wall surrounded by normal
bilaterally in upper lobes, what type of emphysema is the patient suffering from:
a. Centriacinar emphysema
b. Obstructive emphysema
c. Panacinar emphysema
d. Paracicatricial emphysema
e. Paraseptal emphysema
Dahnert Radiology Review Manual 6th edition: 492
36. A previously healthy adult male has been diagnosed with active pulmonary tuberculosis. What imaging
feature would suggest primary rather than reactivated pulmonary tuberculosis?
a. Bilateral reticulo-nodular shadowing
b. Cavitating pneumonia with left hilar lymph nodes
c. Fibrosis and distortion of lung architecture
d. Lingular consolidation with left hilar lymph nodes
e. Tuberculoma formation with pleural effusion
David Sutton 7th edition: 141
37. A 45 year old lady presented with cough and shortness of breath. She also complained of arthalgia. Her
chest radiograph revealed bilateral symmetrical lobulated hilar masses. What is the most likely
diagnosis?
a.Lymphoma
b.Metastasis
c. Pneumoconiosis
d.Sarcoidosis
e.Tuberculosis
David Sutton 7th edition: 187
38. A 60 year old man with Hodgkin’s disease and hypertension complains of shortness of breath. He was
further investigated by chest x-ray which revealed bilateral pleural effusions with clear lung fields.
Which one of the following drug is the most likely cause of his pleural effusions?
a. Amoxicillin
b. Bleomycin
c. Frusemide
d. Lisinopril
e. Propranolol
David Sutton 7th edition: 209
39. A young male after a road traffic accident complained of pleuritic chest pain and shortness of breath.
His initial supine x-ray chest showed multiple rib fractures with suspicion of pneumothorax. Which one
of the following sings not seen on the initial supine film will predominate in an erect PA chest
radiograph?
a. A deep left costophrenic recess laterally
b. Left apical transradiancy and pleural line
c. Undue clarity of the left mediastinal border
d. Unilateral left lung transradiancy
e. Visualization of the undersurface of the heart
David Sutton 7th edition: 93
40. A 60 years old female patient, non-smoker had a routine chest examination which showed a solitary
peripheral mass, what is the most common radiological sign associated with this mass
a. Air-bronchogram on HRCT
b. Calcification in periphery of the mass
c. Smooth margin
d. Sub-pleural mass
e. Upper lobe distribution
th
Wolfgang Dahnert 6 edition 475
41. A 65 year old gentleman presented with right arm pain, wasting of small muscles of hand, anhidrosis
and ptosis of the right eye, what is the most characteristic finding on chest x-ray of this patient
a. Calcified mediastinal mass
b. Cavitating hilar mass
c. Hilar mass with elevated diaphragm
d. Peripheral mass with effusion
e. Right upper lobe mass
Wolfgang Dahnert 6 edition 476 & Sutton 7th edition 110
th
42. On CXR of one year old boy done for immigration, mediastinal mass was seen; your supervisor
reported it as normal thymus, what is the most characteristic sign for normal thymus
f) Lack of calcification in the mass on plain CXR
g) Lack of pressure effects on trachea on plain CXR
h) Lateral margin shows undulation on plain CXR
i) Normal vessels in the mass on plain CXR
j) Significant space-occupying lesion on plain CXR
7th edition Sutton Page 248
43. A young asymptomatic male had an x-ray chest for Canada immigration, radiologist reported a small
right hemi thorax with a soft tissue opacity partially obscuring the right heart border in retro-sternal
location shaped like a Turkish sword: what is the most consistent feature of this anomaly;
a) Absent IVC and anomalies of the bony thorax with excessive areolar tissue
b) Absent or small pulmonary artery perfusing the abnormal lung
c) Arterial supply wholly from the thoracic aorta to the abnormal lung
d) Hypoplasia or aplasia of one or more lobes of the right lung
e) Partial anomalous pulmonary venous return from the abnormal lung
th
7 edition Sutton page 251.
44. A full term baby boy developed respiratory distress, serial x-ray chest done on day 5 shows over
distended left upper lobe which is hypovascular and herniated across the midline with shift of
mediastinum towards the right side. What is the most characteristic feature of this condition:
a. Associated with polycystic kidney disease
b. Involved lobe appears hyper vascular
c. Progressive alveolar wall destruction
d. Progressive over distension of the lobe
e. The lower lobes are frequently involved
7th edition Sutton page 251
45. An infant with few symptoms had an x-ray chest, multiple small nodules of 2-3 mm in size were
scattered throughout the lung fields, what is the most likely cause of this appearance?
a.
Aspiration pneumonia
b.
Bronchial spread of mycobacterium
c.
Immune deficiency syndrome
d.
lymphatic spread of mycobacterium
e.
Venous spread of mycobacterium
th
7 edition Sutton page 260
46. A two year child presented in emergency room with stridor and barking cough for six hours, no history
of fever, x-ray chest show steeple-shaped trachea.
The commonest cause of this condition is:
a) Intubation
b) laryngo-tracheobronchitis
c) Fibrosing mediastinitis
d) Wegener’s granulomatosis
e) Tuberculosis
7th edition Sutton page 161(answer: laryngo-tracheobronchitis, C1, easy, essential)
47. A 30 year old gentleman with history of road traffic accident was brought to emergency department
with GCS of 6/15, he had intra cranial hemorrhage with fracture of parietal bone, and 36 hours later x-
ray chest had diffuse air-space shadowing which remained static for the next one week.
The most likely cause of this diffuse air-space shadowing is:
a) Aspiration pneumonia with pulmonary edema
b) Damaged capillary bed and development of hyaline membrane
c) Multiple drug resistant infection with pulmonary edema
d) Non-Cardiogenic pulmonary edema due to fluid overload
e) Superadded bacterial infection with pulmonary edema
7th edition Sutton page 243(answer: Damaged capillary bed with development of hyaline membrane, C2, moderate,
important)
48. A 40 year lady presented with dry cough for two days. She has received chemotherapy and radiation for
right breast cancer one year back. On assessment she is tumor free, however new finding was reported
on her CXR.
The most common likely finding in chest after 9-12 months of treatment is:
a) Development of sarcoma
b) Non healing fractures
c) Osteonecrosis
d) Pleural effusion
e) Pulmonary fibrosis
th
7 edition Sutton page 236 (answer: Pulmonary fibrosis, C1, easy essential)
MSK:
1. Avascular necrosis
2. Paget’s
3. factor VIII deficiency
4. trauma spine
1. A 40 years lady, known case of Systemic Lupus Erathamatosis on regular treatment, started to have left hip pain
with limited movements for 4 days. No history of trauma or surgery. Her treating physician advised. MRI of left
hip joint.
MRI revealed low signal intensity band with sharp inner surface + blurred outer margin on T1 weighted
sequences.
a. Avascular necrosis
b. Bony metastasis
c. Epiphyseal fracture
d. Gout
e. Spondyloarthropathy
(Dahnert 6th edition page 49-50)
2. A 65 years gentleman consulted his family physician for increasing size of his hat in the last one year. He has no
co-morbid. His physician advised skull X-ray. The skull X-ray showed mixed lytic and blastic pattern of thickened
calvarium giving “cotton-wool” appearance.
The most characteristic appearance of this disease on plain x-ray is:
3. A 3 years baby boy presented with bilateral knee joint swelling. He has family history of bleeding disorder. His
mother does not remember any significant trauma. On blood analysis he is found to have factor VIII deficiency.
In early stage of the disease this swelling most likely would be because of:
5. A 38 years old lady is brought to ER after suffering from road traffic accident. She was passenger of car in which
3 persons were sitting and 2 of them died at the spot. She has slight pain in her neck and abdominal pain. The
ER team after stabilizing her orders radiological investigations.
The set of first four radiological investigations would be:
a. X-ray chest AP, X-ray abd AP, X-ray C-spine Lat Shoot through & ultrasound FAST (Focused Assessment
with Sonography in Trauma)
b. X-ray pelvis AP, X-ray abd AP, X-ray C-spine Lat Shoot through & ultrasound chest
c. X-ray pelvis AP, X-ray abd AP, X-ray whole spine AP & ultrasound FAST
d. X-ray pelvis AP, X-ray chest AP, X-ray C-spine Lat Shoot through & ultrasound chest
e. X-ray pelvis AP, X-ray chest AP, X-ray C-spine Lat Shoot through & ultrasound FAST
Answer: e
7. A 27 year old lady under MRI lumbar spine to evaluate her back pain which was radiating to her left lower limb.
The pain was sharp in nature and was aggravated by movements. On MRI spine there was no definitive disc
disease noted, however there was incidental finding of mottled pattern of low to high on T1 weighted images
and very high signal intensity on T2 weighted sequences of L2 vertebra giving a characteristic “corduroy” sign.
On post contrast this lesion will enhance because of:
a. Avascular necrosis
b. Hypervascularity answer: b
c. Hypovascularity
d. Infective nature
e. Malignanct nature
(Dahnert 6th edition page 99)
8. A 30 year old male patient who is cirrohitic, having bronze diabetes with hypogandism and decrease libido
underwent X-ray hand to evaluate back pain. The characteristic appearance seen on this X-ray would be
a. Chondrocalcinosis with degenerative changes
b. Hook like osteophytes on radial aspect of metacarpal heads
c. Punched out lesions with soft tissue component answer: b
d. Radiating trabecular thickening
e. Soft tissue mass with phelboliths
(Dahnert 6th edition page 101)
9. A 30 years old gentle man, deep sea diver by profession had to rapidly ascend from bottom of the ocean
because of some fault in his equipment. He started to experience paresthesias and substernal discomfort with
coughing.
In bones the healing phase of this disease is represented by:
10. A 5 year boy presents with swelling around the knee after a fall from chair. Mother also gives history of
low-grade fever, fatigue and easy bruising with minor injuries for last two months. Plain x-rays of knee
show metaphyseal translucent band in the lower end of femur and upper end of tibia.
The child is most likely suffering from:
a) Acute-lymphoblastic leukemia
b) Langerhans cell histiocytosis
c) Lead poisoning
d) Rheumatoid arthritis
e) Tuberculous arthritis
Reference: David Sutton 7th edition (Haematopoietic disorder) 1327, chapter 41 (b), Dahnert 6 th
edition 112 (a)
11. An asymptomatic healthy teenager had routine x-ray chest PA and lateral view. On chest x-ray his lungs
and heart were unremarkable. In comparison with previous x-ray of one year back no interval change was
noted in the partial wedging of T10-T12 vertebrae with Schmorl’s nodes and disc space narrowing.
The most likely diagnosis is:
a. Calve’s disease
b. Developmental vertebral notching
c. Osteochondrodystrophy
d. Scheuermann disease
e. Tuberculous spondylitis
Reference: David Sutton 7th edition: 1187-88(C1, easy, important)
12. A 26 year old gentleman presented to the emergency department with swollen and bruised right hand. He
was involved in a street fight.
The most likely fracture which he sustained is:
a. 1st metacarpal base
b. 5th metacarpal shaft
c. 5th proximal phalanx base
d. Scaphoid waist
e. Triquetral body
Reference: David Sutton 7th edition: 1410 (C1, easy, essential)
13. A 60 year old diabetic lady presented with swelling of left foot for three days. She was also suffering from
urinary tract infection for two weeks. Her clinician suspected osteomyelitis.
The earliest radiological features seen on plain x-ray to suggest osteomyelitis are:
a. Detached necrotic cortical bone
b. Endosteal erosion
c. Intracortical fissuring
d. Localized soft tissue swelling
e. Speculated periosteal reaction
Reference: Dahnert Radiology Review Manual 6th edition: 138 (C1, easy, essential) d
14. A middle age gentleman had plain x-ray of hand for pain. On x-ray joint space narrowing was noted in
distal inter phalangeal joints with radial sublaxation of first metacarpal base.
The most likely diagnosis is:
a. Gouty arthritis
b. Osteoarthritis
c. Psoriatic arthritis
d. Reiter’s arthritis
e. Rheumatoid arthritis
Reference: Dahnert Radiology Review Manual 6th edition: 15 (C2, easy, essential)
15. A 30 year old gentleman falls on his out-stretched hand. On plain lateral view of the wrist a chip of bone is
identified on dorsal aspect of the carpaus.
The most likely bone involved in the fracture is:
a) Hamate
b) Lunate
c) Pisiform
d) Scaphoid
e) Triquetrum
Reference: 7th edition Sutton p 1408 (C1, easy, essential)
16. A 10 year old female child presented with progressive painless swelling and deformity of the right hand. Her
plain x-ray of hand showed multiple round lytic lesions with ground glass appearance and calcification. No
cortical break noted.
The most likely diagnosis is:
a. Lichtenstein-Jaffe disease
b. Maffucci syndrome
c. Mazabraud syndrome
d. Ollier disease
e. Trevor disease
Reference: 7th edition Dahnert page 73 (C2, easy, essential) d
17. A 45 year old lady presented with arthalgia involving both hands and fatigue for three months. She has multiple
findings on plain film.
The most characteristic radiological finding to differentiate psoriasis from rheumatoid arthritis is:
a) Erosions
b) Joint sublaxation
c) Normal bone density
d) Periosteal reaction
e) Soft tissue swelling
Reference: David Sutton 7th edition page 1216 (C1, easy, essential)
18. A 10 year old boy has severe left leg pain for 6 months. The pain worsens at night. His plain x-ray of left femur
showed an area of sclerosis on proximal diaphysis.
The most likely diagnosis is :
a) Bone island
b) Ewing’s sarcoma
c) Osteoblastoma
d) Osteogenic sarcoma
e) Osteoid osteoma
Reference: _4th edition Chapman and Nakielny page no: 586
19. A 35 year old construction labor sustained face injuries after fall from 20 feet height. His CT scan of skull and
face show, unilateral fracture line passing through right posterior alveolar ridge, medial orbital rim and across
nasal bones with involvement of pterygoid process. The most likely diagnosis is :
a) Blowout fracture
b) Fracture petrous bone
c) LeFort I fracture
d) LeFort II fracture
e) LeFort III fracture
Reference: 7th edition Dahnert page 208
20. A 40 year old gentleman presented with swelling over right clavicle .An aggressive lesion was noted on plain x
ray involving right clavicle. He has past history of receiving chemotherapy and radiation for right side neck
lymphadenopathy due to non-Hodgkin’s lymphoma.
The most likely lesion in the clavicle :
a) Chondrosarcoma
b) Fibrosarcoma
c) Malignant fibrous histiocytoma
d) Osteosarcoma
e) Plasmacytoma
Reference: 7th edition Dahnert page 150
21. A 70 year old gentleman had a fall on outstretched hand .He had a Colle’s fracture on plain x ray. In addition to
this fracture he had shaggy periosteal reaction around the distal third of radius and ulna. On further inquiry
there was significant history of weight loss in the last one year.
The most likely cause of shaggy periosteal reaction in this case is:
a) Bronchiectasis
b) Bronchogenic carcinoma
c) Fibrous mesothelioma
d) Heart disease
e) Tuberculosis
Reference: David Sutton 7th edition page 1231
22. An adolescent male complained of pain and swelling of right knee joint, on imaging your supervisor gave the
diagnosis of “pigmented villonodular synovitis”.
The imaging feature most characteristic of this condition is:
24. A 15 year boy presents with sever localized pain in right thigh for two weeks. He also has fever.
Laboratory investigations show anemia and leukocytosis. Plain x-rays of right femur are done.
The radiological feature favoring Ewing’s sarcoma is:
a) Cortical thickening
b) Hair-on-end appearance
c) Moth-eaten appearance
d) Penetration in soft tissue
e) Sclerotic new bone formation
Reference: David Sutton 7th edition (tumor and tumor like conditions, Ewing’s sarcoma) 1315,
chapter 40 (b), Dahnert 6th edition 75
25. A 65 year lady presents with compression fracture of D11 and D12. Coarse trabecular pattern is noted on
plain x-rays in rest of the spine. In this age group clinician wants differentiate between osteoporotic from
metastatic disease to spine.
The next most appropriate step is:
26. A two weeks old baby girl presented with painful right hip. She was born at home by assistance of a mid-
wife. Her x-ray hip shows gross metaphyseal destruction, cortical erosions and involucrum involving the
entire femur.
The most likely organism responsible for this appearence is:
a) Brucella
b) Listeria
c) Mycobacterium tuberculosis
d) Streptococcus
e) Treponema pallidum
Reference: David Sutton 7th edition (bone and joint infections) 1164, chapter 36 (d)
27. After sustaining a road traffic accident patient was brought to ER with bleeding from left ear .On examination
no neurosensory hearing loss. The most characteristic feature to be expected on CT scan is :
f. Facial nerve injury due to fracture of facial canal
g. Fracture line is parallel to the axis of the Petrous pyramid
h. Fracture line perpendicular to axis of Petrous pyramid
i. Fracture of the molar tooth on sagittal CT scan
j. Incudostapedial joint dislocation
Reference: Dahnert Radiology Review Manual 6th edition: 208-9
28. A young male 20 years of age suffers hyper flexion injury of cervical spine while participating in a motorcycle
race. On plain films he is diagnosed to have an un-stable fracture of cervical spine.
The most likely type of fracture is:
a) Anterior sublaxation of C3 over C4
b) Flexion instability at C3 level
c) Flexion-tear drop fracture of C6
d) Odontoid fracture
e) Spinous process fracture of C7
Reference: 7th edition Dahnert page 209
1. A 30 year old lady presents with fever and palpitations. Lab findings of the patient show an increased TLC,
raised ESR and raised serum free T3, T4. She was advised a Thyroid scan on which there was a non visualization
of the thyroid gland and a good visualization of the salivary glands. The most likely diagnosis is:-
a. Agenesis of the Thyroid gland
b. Graves disease of the thyroid
c. thyroiditis
d. Blocked gland secondary to amiodarone intake (high serum T3 t4 also seen here)
e. Hasimoto’s thyroiditis
Reference: Dahnert 6th edition, page: 402
2. A 20 year old male, a rising cricket bowler, has recently developed pain in his right leg. His doctor ordered a
three phase skeletal scan which ultimately revealed an increase in the perfusion blush, blood pool and a focal
fusiform uptake in the mid part of the posterior aspect of the right tibia. The most likely explanation for this
patients findings is:
a. Cellulitis
b. Osteomylitis
c. Stress fracture
d. Osteoid osteoma
e. Aneurismal bone cyst
Reference: Sutton, 7th edition, page: 1375
3. A 40 year old lady with sever generalized bony pains was evaluated and found to have a multinodular goiter.
Her labs showed a normal Thyroid profile but a raised serum PTH. Her Parathyroid Sestamibi scan was positive
for a functional adenoma at the lower pole of the left lobe of the thyroid. The most appropriate next step in
the diagnosis is:
a. Three phase bone scan
b. Ultrasound of the thyroid
c. MRI of the thyroid
d. Technetium scan of the thyroid
e. CT of the neck
Reference: Sutton 7th edition, page: 69
4. A 10 year old boy, while playing hockey suffered a hit on his leg with the opponent’s hockey stick. After two
days he developed pain and swelling over the injury. He was advised an x-ray that showed no fracture. He was
then advised a bone scan that showed increased perfusion blush and blood pool uptake. The 3 hours delayed
scan was within normal limits. The most likely explanation for this patients finding is:
a. Osteomylitis
b. Cellulitis
c. Clellulitis with underlying bony involvement
d. Enthesiopathy
e. Malignant process
Reference: Sutton, 7th edition, page: 1426
CNS + H&N
1. Juvenile angiofibroma
2. left basal ganglia and internal capsule infarct
3. NF1
4. Hyperparathyroidism
5. Hypercementosis
6. Fungal infection in PNS
7. Polyposis
8. Imaging of intracerebral hematoma
9. Neck tumor extent
10. Hamartoma tuber cinerum
11. Dentigerous cyst
12. Graves ophthalmopathy
13. PilocyticAstrocytomas
14. Carpenter’s syndrome
15. Tuberous sclerosis
16. Myelomeningocoele
17. Imaging protocol of stroke
18. Meningioma
19. Tumor of the pineal gland
20. Hippocampal sclerosis
21. Hemimegalencephaly
22. Vocal cord tumor
23. PNS carcinoma
24. Paget’s disease
25. Ameloblastoma
26.Retinoblastoma
27.Persistent Hyperplasic Primary Vitreous
28.Glioblastoma multiforme
29.
1. A 15 year old male presented with severe epistaxis and nasal speech.
His clinician suspects juvenile angiofibroma. The most characteristic imaging feature on
CT scan is:
a. Anterior bowing of posterior antral wall
b. Highly vascular mass
c. Invasion of sphenoid sinus
d. Widening of orbital fissure
e. Widening of pterygopalatine fossa
Reference: Dahnert Radiology Review Manual 6th edition: 387
2. 20 year old boy was brought to ER with complaints of severe backache and bladder incontinence for
last 2 months. On examination surgeon noticed gait abnormality and some sensory/motor deficit as
well. MRI spine was carried out that showed an ill defined heterogeneously enhancing eccentric mass
lesion within the upper thoracic region of spinal canal with associated cord expansion with meningeal
enhancement . No significant surrounding edema seen.
Most likely diagnosis is:
a) Astroctytoma
b) Ependymoma
c) Hemangioblastoma
d) Meningioma
e) Metastasis
Key - a
3. 65 year old gentleman presented to neurosurgery clinic with complains of headaches off and on,
vomiting and left arm weakness for 6 months. MRI brain was performed that revealed a large
heterogeneous mass lesion in the right frontal lobe showing peripheral contrast enhancement and
extension across midline. Significant surrounding edema exceeding the tumor volume also seen.
The most appropriate diagnosis based on MRI findings is:
a) Glioblastoma Multiformes
b) Lymphoma
c) Meningioma
d) Metastasis
e) Tuberculous abscess
Key- d
4. A 76 years gentleman presented with right sided weakness since last 4 days. He is a known diabetic
for 20 years. He underwent MRI brain for further evaluation.
The most characteristic finding on MRI will be:
a. High on Diffusion Weighted image in right MCA territory
b. Intravascular enhancement sign in left MCA territory
c. Low on Diffusion Weighted image in left internal capsule
d. Low signal on ADC image in right basal ganglia
e. Meningeal enhancement on the right side
Reference: Dahnert 7th edition, page 299-301
5. A 65 years gentleman presents in neurology clinic with gait instability, urinary incontinence, and
dementia. His physician orders MRI brain.
The most characteristic finding on MRI will be:
a. Cerebral and cerebellar atrophy with ventricular dilatation
b. Normal periventricular white matter signals on FLAIR
c. Prominent sylvian fissures with ventricular dilatation
d. Rounded gyri against calvarium on all sequences
e. Ventricular dilatation with normal brain parenchyma
Reference: Dahnert 6th edition, page 298-9, Sutton 7th edition, page 1794(e)
6. A 5 year girl presents with progressive loss of vision in the last 6 months. On examination scoliosis of
spine was noted along with café-au-lait spots on skin. For further assessment she underwent MRI
brain.
The characteristic finding seen on MRI in orbit is:
f. Calcification in optic nerve
g. Homogenous contrast enhancement
h. hyperostosis of lateral wall of orbit
i. kinking of the optic nerve
j. normal size optic canal
Reference: Dahnert 7th edition (optic nerve glioma) 348 (d)
7. A middle aged lady complained of generalized body aches, weakness and loosening of
teeth for 4 months. Her OPG showed generalized resorption of lamina dura and one
centimeter size well defined lytic lesion at the angle of jaw.
The pathognomonic feature related to this condition:
a) Elevated serum Calcium Level
b) Raised serum alkaline phosphatatse
c) Soft tissue calcifications
d) Subperiosteal bone resorption
e) Widespread lytic lesions
Sutton 7th Edition, page 1539 and Dahnhert 6th Edition, page 104-106 (d)
8. X-ray mandible of an adult shows very dens teeth especially at the apices and body of
the roots with bulbous expansion. The lamina dura and periodontal membrane are
intact and there are no surrounding lucencies.
The most likely diagnosis is:
a) Cementoma
b) Complex odontome
c) Compound odontome
d) Hypercementosis
e) Postinflammatory sclerosing osteitis
Sutton 7th Edition, Chapter 49, page 1535 (d)
9. A 6 year boy presented with facial swelling. He has received two cycles of
chemotherapy for lymphoma. His clinician suspects fungal infection and advises MRI
paranasal sinuses.
The radiologist confirms fungal infection on the basis of:
a) Absence of signal on all sequences
b) Extension of disease intracranially
c) High signal on T2 weighted images
d) Involvement of sphenoid sinuses
e) Low signal on T1 weighted images
Sutton 7th Edition, Chapter 48, page 1523 (fungal infection PNS) (a)
10.A middle aged man presented to ENT clinic with complaints of recurrent episode of
nasal blockage and sneezing. CT was done for assessment of PNS. CT scan shows soft
tissue opacification of all paranasal sinuses and nasal cavity, widening of infundibulum,
and thinning of sinus walls
The most likely diagnosis is:
a) Carcinoma
b) Fungal infection
c) Mucoceles
d) Pan Sinusitis
e) Polyposis
Sutton 7th Addition pg 1521(e)
11. A 76 years gentleman presented with right sided weakness since last 4 days. He is a known diabetic
for 20 years. He underwent MRI brain for further evaluation.
The most characteristic finding on MRI will be:
k. High on Diffusion Weighted image in right MCA territory
l. Intravascular enhancement sign in left MCA territory
m. Low on Diffusion Weighted image in left internal capsule
n. Low signal on ADC image in right basal ganglia
o. Meningeal enhancement on the right side
Reference: Dahnert 7th edition, page 299-301
12. A 78 year gentleman is found collapsed in his bedroom. His family gives history of hypertension for
last 25 years. His initial CT demonstrates a large intracerebral hematoma; the mean CT attenuation is
70 HU, with fluid / fluid levels.
The most likely age of hematoma is:
p. 0 – 2 hours
q. 2 – 4 weeks
r. 2 -3 months
s. 3 – 48 hours
t. 3 – 7 days
Reference: Dahnert 6th edition, page 292-295
13.A 45 year old gentleman underwent enhanced CT scan of neck. The soft tissue planes
medial to the styloid process on left side were obliterated. Fossa of Rosenmuller was
also obliterated. Enhancing enlarged lymph nodes were noted at level II, II and IV on
left side.
The most likely clinical presentation of this patient is
a) Difficulty in eating
b) Discharge with deafness
c) Fever and cough
d) Neck pain and rigidity
e) Sore throat
Reference: seventh edition Sutton page 1492. 6th edition Dahnert page 395(b)
14. A two year old male child presented with gelastic seizures and precocious puberty since 3 months, he
also had neurodevelopmental delay.
The most appropriate imaging to diagnose his condition is:
a) CT abdomen with renal protocol
b) MAG-3 nuclear scan for kidneys
c) Plain x-rays for bone age estimation
d) Ultrasound for adrenal glands
e) With and without contrast MRI brain
Reference Wolfgang Dahnert 6th edition page: 299
15. A young adult male underwent OPG for trauma .He was found to have an expansile radiolucent lesion
in close relation to the crown of the lower third molar. The most likely diagnosis is :
a. Aneurysmal bone cyst
b. Dentigerous cyst
c. Developmental cyst
d. Odontogenickeratocyst
e. Radicular cyst
Reference: David Sutton 7th edition: 1543
16. A 40 year old lady presented with proptosis, eyelid retraction and sclera injection She underwent MRI
orbits, which revealed massive enlargements of medial rectus bilaterally with sparing of tendinous
insertion.
The most likely diagnosis is:
a. Carotid cavernous fistula
b. Graves ophthalmopathy
c. Lymphoma
d. Orbital pseudotumor
e. Rhabdomyosarcoma
Reference: David Sutton 7th edition: 1579
17. A 2 year old child was investigated for mentalretardation. He had microcephaly, obesity and
polydactyly .He underwent x-ray skull which showed Craniosynostosis.
He is most likely suffering from:
a. Achondroplasia
b. Apert’s syndrome
c. Carpenter’s syndrome
d. Crouzon’s syndrome
e. Neurofibromatosis
Reference: David Sutton 7th edition: 1624
18. All are true about pituitary stalk except:
a. Arises from the floor of the third ventricle
b. Formed from axons of cells in hypothalamus
c. Joins posterior lobe at its junction with anterior lobe
d. Poor contrast enhancement on dynamic scanning
e. Up to three millimeter thick superiorly in adults
Reference: Dahnert Radiology Review Manual 6th edition: 254
19. A 6 years old female child presented with headache, vomiting and gait disturbance .Her MRI brain
showed a cystic lesion in the cerebellum with intensely enhancing mural nodule compressing the
fourth ventricle resulting in non-communicating hydrocephalus. The most likely diagnosis is :
a. Atypical Medulloblastoma
b. Ependymoma
c. Hemangioblastoma
d. Metastasis
e. Pilocytic Astrocytomas
Reference: Dahnert Radiology Review Manual 6th edition: 271
20. A 2 year old male child presented with myoclonic seizures, had mental retardation and brown patches
over the body. His elder sibling died at the age of 6 years with similar complaints. His MRI brain
showed hyperintense T2 lesions in sub-ependymal location. The most likely diagnosis is :
a. Ataxia Telangiectasia
b. Neurofibromatosis
c. Sturge-Weber syndrome
d. Tuberous sclerosis
e. Von-HippleLindau disease
Reference: Dahnert Radiology Review Manual 6th edition: 331
21. A full-term 2.4 Kg boy is born with Myelomeningocoele .He came to your department for imaging of
brain and spine .The most consistent feature in brain in 95 % of cases is:
a) Absence of falx and tentorium cerebelli
b) Cerebellar hypoplasia with brainstem herniation
c) Cervical or occipital encephalocele
d) Enlargement of cerebellar vermis
e) Partial/complete agenesis of Corpus callosum
Reference: David Sutton 7th edition page 1728, 7th edition Dahnert page 275
22. A 60 year old female known case of diabetes mellitus presents in ER with weakness of the right lower
limb for four hours .Her initial un-enhanced CT scan is negative .The imaging protocol most helpful in
the patient management is :
a) Diffusion weighted and dynamic perfusion protocols
b) Routine brain sequence with and without contrast
c) Routine brainsequence, diffusion & perfusion protocols
d) Routine brain sequence, diffusion, perfusion without contrast
e) Routine brain sequence, FLAIR and MRA
Reference: David Sutton 7th edition page 1823
23. A middle age female undergoes plain films of the skull after road traffic accident. She has a linear
parietal bone fracture in addition to ball-like amorphous calcification in para- sagittal location on right
side. The most likely cause of this calcification is :
a) Chordoma
b) Craniopharyngioma
c) Germinoma
d) Meningioma
e) Oligodendroglioma
Reference: David Sutton 7th edition page 1629
24. The Houns Field unit of normal grey matter is:
a) 0
b) 22-32
c) 36-46
d) 56-76
e) 80-1000
Reference: David Sutton 7th edition page 1724
25. 9 years old boy presented with precocious puberty .On examination he has Parinaud syndrome
(paralysis of upward gaze).The most likely finding on MRI of the brain with contrast is:
a) Well-circumscribed tumor of the hypothalamus
b) Well-circumscribed tumor of the pes-hippocampus
c) Well-circumscribed tumor of the pineal gland
d) Well-circumscribed tumor of the pituitary gland
e) Well-circumscribed tumor of the tuber cinereum
Reference: 7th edition Dahnert page 321
26. A 5 year old boy is regularly attending a neurology clinic for partial epilepsy of habitual type for the last
one year.
The most likely finding to be seen on his MRI brain is:
a) Benign neoplasm
b) Cortical malformations
c) Cortical scar
d) Hippocampal sclerosis
e) Vascular lesions
Reference: David Sutton 7th edition page 1804
27. A 12 months old baby boy presents with fits and hemiplegia since birth. On examination he has
unilateral skull vault enlargement.
The most likely cause of his signs and symptoms is:
a) Alexander’s disease
b) Familial type large skull
c) Hemimegalencephaly
d) Hydrocephalus
e) Spongy degeneration
Reference: Reference David Sutton page: 1728
28. A 60 year old lady singer underwent enhanced CT scan of neck. Her presenting complaints were
hoarseness of voice for the last three months. She gives history of smoking 30 cigarettes per day for
the last 40 years. On the basis of CT findings her clinician advises her for total laryngectomy.
The most likely reason for this advice is:
a) Deep lateral extension into paralaryngeal space
b) Enlarged lymph nodes in the deep cervical chain
c) Involvement of thyroid and arytenoids cartilages
d) Involvement of the anterior commissure
e) Tumor confined to vocal cord with normal mobility
Reference: seventh edition Sutton page 1498. 6th edition Dahnert page 387-88(c)
29.A 45 year gentleman underwent enhanced CT of PNS for staging of Squamous cell
carcinoma in right maxillary sinus. His CT shows destruction of posterolateral maxillary
wall with extension of the disease.
The extension is into:
a) Infratemporal fossa
b) Nasal cavity
c) Paraphyrangeal space
d) Pterygomaxillary fossa
e) Pterygopalatine fossa
Sutton 7th Edition, Chapter 48, page 1526 (Squamous cell carcinoma), (a)
30.A 70 years gentleman visited a dental surgeon for right sided tooth ache of sharp
character radiating along the lower jaw. The dentist could not find any obvious signs of
dental pathology on examination, except for few missing teeth. He advised an OPG
which showed significantly enlarged and patchy sclerotic entire mandible.
Further advice of the dentist will be:
a) Consult metabolic bone specialist
b) Partial mandiblectomy
c) Repeat OPG every 3 months
d) To have a bone scan
e) To undergo bone biopsy
Sutton 7th Edition, Chapter 49, page 1536, Dahnhert 6th Edition, page 146 (for treatment)
(a)
31.Middle age man presented with slow growing painless swelling of the jaw. X ray
showed 3 cms size, well defined, expansile, multilocular lytic lesion with an unerupted
tooth, at the angle of mandible. The lesion was excised but several months later it
recurred and was larger than before.
The most likely diagnosis is:
a) Ameloblastoma
b) Dentigerous cyst
c) Odentogenic keratocyst
d) Radicular cyst
e) Simple bone cyst
Sutton 7th Edition, Chapter 49, page 1545 and Dahnhert 6th Edition, page 181(a)
32.A 60-year-old gentleman presented with sudden loss of vision. He is a known diabetic
for last 15 years. He is referred for an ultrasound of the eye.
The most appropriate ultrasound technique is:
a) 2 dimensional B scan
b) Color Doppler
c) Motion mode
d) One dimensional A scan
e) Time Motion mode
David Sutton 7th edition: 1551 (a)
33.A 3 months old baby girl presented with unilateral micropthalmia and white pupillary
reflex in the same eye. On ultrasound, an echogenic focus in the retrolental region is
noted.
The most likely diagnosis is:
a) Coats Disease
b) Persistent Hyperplasic Primary Vitreous
c) Retinoblastoma
d) Retinopathy of prematurity
e) Toxocariasis
David Sutton 7th edition: 1557 (b)
34.A 10-year-old boy sustained a projectile injury to the right eye while playing at his
father’s workshop. His ultrasound of the eye was unremarkable. However due to
clinical suspicion he underwent examination under general anesthesia and a foreign
body was removed.
The likely cause of negative ultrasound was:
a) Metal
b) Glass
c) Stone
d) Wood
e) Air bubble
David Sutton 7th edition: 1566 (d)
35.Parents of a two year old girl noticed decrease vision in left eye. On eye examination,
she has white pupilary reflex in her left eye. Calcification was noted on eye ultrasound
The most likely diagnosis is:
a) Retinal dysplasia
b) Retinopathy of prematurity
c) Coat’s disease
d) Retinoblastoma
e) Toxocariasis
Aids to Radiological Differential Diagnosis by Chapman and Nakielny 3rd edition.364 (d)
36. A 27 year gentleman presents with waxing and waning retrobulbar pain, loss of vision
and ataxia for 3 months. He undergoes enhanced MRI brain and spine.
The classic finding on MRI for the likely diagnosis will be:
a) Atrophy of the corpus callosum
b) T2 bright signals in cerebellum
c) Cortical central atrophy
d) Peri-callosal T2 bright signals
e) T2 bright signals in cord
Reference: Dahnert 6th edition, page 310-11(d)
37.A 68 year gentleman presented with headaches for 6 months and sudden loss of
consciousness for 2 hours. He undergoes MRI brain. MRI shows poorly defined lesion
with some mass effect and vasogenic edema.in right frontal lobe. The lesion is crossing
the midline.
The most likely diagnosis is:
a) Ganglioglioma
b) Glioblastoma multiforme
c) Grade I astrocytoma
d) Metastasis
e) Oligodandroglioma
Reference: Dahnert 7th edition, page 288-9
1) A 40 yrs old male presents with pain in the jaw associated with dental caries. An OPG demonstrates a lytic
lesion of 1 cm in diameter, associated with the apex of a carious tooth. The lesion has a thin rim of cortical
bone and there is some resorption of the root.
The likely diagnosis is
a) Ameloblastoma
b) Dentigerous cyst
c) Odontogenic keratocyst
d) Odontoma
e) Radicular cyst
2) A 42 yrs old male presents with a painless mass involving the right side mass the mandible. Radiographers
show a well defined lucent lesion. Which of the following additional findings favor a diagnosis of
Ameloblastoma over a Radicular cyst?
a) absence of matrix mineralization
b) location at the symphysis menti
c) location at the root of a tooth
d) soap bubble appearance
e) rim of cortical bone
3) A 24 yrs old male presented in clinic with H/O bleeding P/R, abdominal pain and facial pain. His barium
enema revealed multiple polypoid lesions in colon. His skull X-rays showed well defined sclerotic lesions in
mandible and skull.
The most likely diagnosis is:
a) cementomas
b) fibrous dysplasia
c) osteomas
d) osteoclastoma
e) sclerotic metastasis
4) Skull x-rays in a male child demonstrate brachycephaly, wormian bones, large mandible and small upper
jaw with multiple crowded supernumerary teeth.
The syndrome which could explain these findings is:
a) Crouzon syndrome
b) Cleidocranial dysplasia
c) Down’s syndrome
d) Osteopetrosis
e) Pyknodysostosis
5) A 26 yrs old male with learning difficulties presents to emergency department after suffering facial injuries.
An OPG is performed that demonstrates a multilocular radiolucent lesion in the angle of the mandible. He
had suffered from multiple carcinomatous lesions of the skin in childhood.
The most likely diagnosis is:
a) Ameloblastoma
b) Aneurysmal bone cyst
c) Giant cell tumor
d) Multiple myeloma
e) Odontogenic keratocyst
CVS:
1. Bicuspid aortic valve
2. Patent ductus arteriosus
3. Aberrant left pulmonary artery
4. Investigation for endocarditis
5. chronic thromboembolism
6. RTA: chest injuries
7. Cross sectional anatomy of heart
8. Cause leg claudications
9. Ebstein's anomaly
10. Cardiac MRI in MI
11. Interpretation of nuclear scan in thromboembolism
12. ASD
13. aortic dissection
14. aortic regurgitation
15. imaging of cardiac surgery
16. TOF
16. As even year old boy from a Thar presents with cyanosis and fainting spells on exertion. The
frequency has gradually increased. On examination he is on 5th percentile of height and weight and
has clubbing
All except one are constant features of this condition:
a. Aorta arising from both the ventricles
b. Large VSD immediately below aortic valve
c. Left ventricular outflow tract obstruction
d. Pulmonary artery stenosis
e. Right ventricular hypertrophy
Reference: David Sutton 7th edition 382-3
1. A 7 days neonate developed lower extremity cyanosis. The CXR showed an enlarged heart with
increased pulmonary vasculature. The aorta had a figure of "3" appearance.
The most common associated abnormality is:
Reference: David Sutton 7th edition: 380, (C1, easy, nice to know)
2. A 45 years old male with history of night sweats and weight loss was investigated. His Monteux test was
negative, ESR was raised, X-ray chest showed widening of right paratracheal stripe, with clear lung fields; what
is the most likely cause of chest x-ray finding?
a. Mediastinal lymphadenopathy
b. Mediastinal tumors
c. Mediastinitis
d. Pleural effusion
e. Tracheal malignancy
David Sutton 7th edition: 5
3. A 4 year old child with no comorbids presented to emergency room with high grade fever, running nose and
shortness of breath, his x-ray chest showed hyper inflated lung fields, peribronchial cuffing and bilateral mild
pleural effusion: what is the most likely cause of his signs and symptoms?
a. Atypical measles pneumonia
b. Haemophilus influenza
c. Klebsiella pneumonia
d. Rhino virus
e. Tuberculosis
David Sutton 7th edition: 136 and Dahnert Radiology Review Manual 6 th edition; 412
4. A routine examination on a neonate born at term reveals bounding peripheral pulses and a continuous
murmur, loudest under the clavicle.
The most likely diagnosis is:
5. A 29 yrs old gentleman with history of dysphagia underwent a barium swallow and found to have a
smooth anterior indentation on the esophagus.
The most likely cause of this indentation is:
Reference: Sutton 7th Edition, Chapter 49, page 1533, Dahnert 6th Edition, page 626 (C1, difficult,
essential)
7. A 55 yrs lady presented with reduced exercise tolerance and shortness of breath. Her clinical
examination was unremarkable. CXR showed enlarged central pulmonary arteries and subsequent
chest CT confirmed pulmonary artery enlargement and also showed right ventricular dilatation.
The feature in favor of chronic thromboembolism as the cause of her findings is:
8. A 42 yrs gentleman, who had a high speed RTA, presents acutely to ER. He has severe chest pain
radiating to his back and is hemodynamically unstable.
The most common finding on chest radiograph in this patient is:
9. The right heart border is prominent in a frontal chest radiograph. On the lateral chest radiograph, the
anteriosuperior aspect of the heart shadow is prominent. The most likely chamber involved is:
a. Aberrant right subclavian artery
b. Double aortic arch
c. Right atrium
d. Right ventricle
e. Superior vena cava
10. A habitual smoker middle age male suffered from intermittent claudication in both legs. His social
history reveals history of smoking for last 25 years. The most likely site of atherosclerotic disease in
lower limbs on angiography is:
8. A 3 week old baby boy presented with tachypnoea and mild cyanosis since birth. His CXR showed cardiomegaly. An
echocardiogram revealed a dilated right atrium and abnormal tricuspid valve with a small, dysplastic but functioning
right ventricle.
9. A 68 yrs lady was diagnosed with a q-wave acute myocardial infarction and underwent a cardiac MRI the following
day.
The most likely signal intensity of the infracted region on T-2 weighted imaging is:
a) High
b) Isointense
c) Low
d) Signal void
e) Variable
Reference: Dahnert 6th edition, p. 645
10. A 40 yrs lady presented with shortness of breath for few hours. Her CXR was normal. She underwent a
ventilation/perfusion study to investigate a possible pulmonary embolus. This showed two small, unmatched sub
segmental defects in the left apical region. The ventilation images are normal.
a) High probability
b) Intermediate probability
c) Low probability
d) Normal study
e) Very low probability
Reference: Sutton 7th edition, pulmonary vessels, P. 1265
11. A 39 yrs old lady presented with fatigue and dyspnoea to her GP who requested a CXR. This showed an enlarged
heart and pulmonary outflow tracts with a small aorta and pulmonary plethora.
a) Aortic regurgitation
b) Atrial septal defect
c) Ebstein's anomaly
d) Pulmonary stenosis
e) Ventricular septal defectReference:
Reference: Dahnert 6th edition, P. 624-625)
12. 68 yrs old gentleman with tearing chest pain radiating to his back was investigated with a contrast enhanced CT. CT
demonstrated an intimal flap separating the aortic lumen in two separate channels. The flap was seen to originate just
distal to origin of left subclavian artery and to extend into the left common iliac artery.
a) Stanford type 1
b) Stanford type 2
c) Stanford type 3
d) Stanford type A
e) Stanford type B
Reference: Sutton 7th edition, Thoracic aorta, P. 309-11
13. A 36 yrs old gentleman presents with shortness of breath and hemoptysis. There is no leg swelling and ECG
is normal. A chest radiograph shows a triangular, pleural based opacity in right mid zone with an ipsilateral
effusion.
The investigation most helpful in making the diagnosis would be:
a) Conventional pulmonary angiogram
b) CT pulmonary angiogram
c) High Resolution CT
d) Staging CT of chest
e) V/Q scan
Reference: Sutton, 7th edition, Pulmonary vessels, P. 1263-65
14. A 26 yrs old lady with known Marfan’s syndrome presents with chest pain and shortness of breath. An
echocardiogram is performed.
The most likely echo finding is:
a) aortic regurgitation and dilatation
b) aortic stenosis and post stenotic dilatation
c) global myocardial wall thickening
d) pulmonary stenosis
e) ventricular septal defect
Reference: Dahnert, 6th edition, P. 115-116
15. A full term baby girl presents with sever cyanosis and breathlessness on fifth day of life. Her chest x-ray
shows pulmonary plethora and narrow pedicle of heart. D-loop transposition is diagnosed on echo.
Life saving procedure will be:
a) Aortocopulmonary window Shunt
b) Blalock Taussig shunt
c) Fontan Procedure
d) Glenn Shunt
e) Rashkind balloon septostomy
Reference: David Sutton 7th edition 385-6
1. Digital subtraction angiography was carried out in a 60 year gentleman for investigation of
subarachnoid hemorrhage. Immediately after angiography patient developed motor aphasia.
Post angiography MRI brain demonstrated acute infarction at the junction of left parietal and
frontal lobe.
The most likely cause of infarction in this case is:
a. Arterial dissection involving the left internal carotid artery
b. Clot formation around catheter tip with subsequent embolism
c. Dislodgement of arterial wall atheroma resulting in embolus
d. Hypertensive crisis resulting in MCA territory infarct
e. Vasospasm in left middle cerebral artery resulting in infarction
2. A 40 year old male underwent digital subtraction angiography for assessment of subarachnoid
hemorrhage. During procedure he suddenly developed bradycardia and became hypotensive but
remained conscious.
The most likely explanation for his sudden change in condition is:
a. Anaphylactic reaction.
b. Embolic stroke.
c. Intracranial hemorrhage.
d. Myocardial infarction.
e. Vasovagal reaction
3. A 65 year old gentleman presented with bleeding per rectum 8 hours. His bleeding/clotting
profile was unremarkable. He underwent colonoscopy which was not conclusive due to
presence of blood clots in the large bowel.
The next best step in management of this patient is:
a. Barium Enema.
b. CT scan of abdomen
c. Digital subtraction Angiogram
d. RBC tagged scan.
e. Ultrasound abdomen
1. A full-term baby boy presented on 5th day of life with bilious vomiting:
The most likely cause of his symptoms is:
a. Colonic atresia
b. Duodenal atresia
c. Esophageal atresia
d. Gastric atresia
e. Jejuna atresia
Reference: 7th edition Sutton p 852
2. On CT scan what is the most pathognomonic sign distinguishing pleural fluid from ascites?
a. Bare area sign
b. Diaphragm sign
c. Displaced crus sign
d. Interface sign
e. Silhouette sign
David Sutton 7th edition: 97
3. A 35-year-old gentleman underwent ultrasound abdomen for dyspepsia.
His ultrasound revealed a large hyperechoeic mass in the liver. On a subsequent MRI scan,
the mass revealed a central scar-showing considerable contrast enhancement.
The most likely diagnosis is:
a) Fibrolamellar carcinoma
b) Focal nodular hyperplasia
c) Hepatic adenoma
d) Hepatocellular carcinoma
e) Intrahepatic cholangiocarcinoma
Reference: David Sutton 7th edition: 795
4. A 22 months baby girl suffers from jaundice and abdominal pain on and off since she was few weeks.
On ultrasound examination intra and extra hepatic cysts are seen.
According to Todani classification this is type:
a) I
b) II
c) III
d) IV
e) V
5. A one year old girl presents in emergency from interior Sindh. She has non-bilious vomiting for one day. On
examination she is on 5th percentile of height and weight. Her abdomen is distended and she is dehydrated.
The best imaging protocol will be:
a) CT scan with oral contrast
b) Gastrograffin meal examination
c) Non-contrast lower GI study
d) Plain x-ray, barium meal examination
e) Plain x-ray, non-ionic upper GI study
Reference: David Sutton 7th edition, page 855-6
6. A full term baby girl born in Hyderabad presents in your emergency drooling, choking and coughing.
Mother says the baby turns blue when she feeds.
The commonest cause of her presentation is:
a) Esophageal atresia with distal fistula
b) Esophageal atresia with proximal and distal fistula
c) Esophageal atresia with proximal fistula
d) H-type tracheosophageal fistula
e) Isolated esophageal atresia
Reference: Sutton 7th edition, page 842
7. A 50 year old female presented in emergency with acute abdominal distension, vomiting and constipation for
two days. She has history of hysterectomy for fibroids and is known to have asymptomatic gall stones for last
10 years. X-ray abdomen shows multiple dilated small bowel loops, multiple air fluid levels and linear lucencies
in right upper quadrant.
The most likely diagnosis is:
a) Adhesive small bowel obstruction
b) Gall stone ileus
c) Incisional hernia with obstruction
d) Jejunal diverticulosis
e) Paralytic ileus
David Sutton Chapter 22, page 673, 7th Edition (Colonic polyps) (b )
8. A 35 year gentleman presented with history of bleeding P/R for six months with weakness and weight loss.
Colonoscopy demonstrated a 4 cm colonic ulcerative polypoidal mass with innumerable tiny colonic
adenomatous polyps. CT scan of abdomen did not reveal any metastatic lesion.
The most appropriate next step for patient management is:
a) MRI of abdomen for occult metastasis
b) Screening for chromosomal abnormalities
c) Screening of siblings and close relatives
d) Total colectomy with post op chemo.
e) Upper gastro intestinal endoscopy
Reference: David Sutton Chapter 21, page 639, 7th Edition (Colonic polyps) (d)
9. A 45 years lady presented with gradual abdominal distension without obstructive symptoms. She does
not have fever but complains of dull generalized abdominal pain. CT scan of abdomen demonstrated
gross multiloculated high density ascitis, scalloping of liver associated speck of calcification in right
iliac fossa.
Most likely diagnosis is:
a) Abdominal Tuberculosis
b) Disseminated lymphoma
c) Mesenteric panniculitis
d) Ovarian carcinoma
e) Pseudomyxoma peritonei
Reference: David Sutton Chapter 19, page 633, 7th Edition (diseases of peritoneum) (e)
10. A 4 weeks old baby boy presented with non-bilious projectile vomiting for 3 days. He undergoes
ultrasound of the abdomen.
The most characteristic sign consistent with his condition is:
a. Delayed gastric emptying in right lateral position
b. Elongated pylorus > 17 mm with thickened muscle
c. Exaggerated peristaltic wave of stomach
d. Indentation of muscle mass on fluid film antrum
e. Pyloric channel mucosa protruding into gastric antrum
11. A 21 year old female has a history of recurrent aspiration pneumonia for one year. She has fluid level
within the mediastinum on chest x ray. Barium swallow examination showed absence of primary
peristalsis.
The most likely diagnosis is :
a. Chaga’s disease
b. Peptic stricture
c. Primary achalasia
d. Scleroderma
e. Secondary achalasia
Reference: 6th edition Dahnert page 797
12. A front seat passenger in a head on collision was brought to the emergency department. A contrast
enhanced CT abdomen and pelvis showed a liver laceration in segment VI that is 2 cm deep and 8 cm
long.
The appropriate grade of liver injury is:
a. Grade I laceration
b. Grade II laceration
c. Grade III laceration
d. Grade IV laceration
e. Grade V laceration
Reference: 7th edition Sutton p 697
13. A 30 year old gentleman known polycythaemia rubra vera presented with progressive abdominal
distension and jaundice for three weeks. His CT showed ascites, caudate lobe hypertrophy with
mosaic enhancement of the liver.
The most likely diagnosis is:
a. Alcoholic hepatitis
b. Budd-Chiari syndrome
c. Chronic liver disease
d. Hepatocellular carcinoma
e. Liver metastasis
Reference: 7th edition Sutton p 768-9
14. A 39 years lady presented with severe abdominal pain of sudden onset for one day. She has a history
of rheumatic heart disease and developed atrial fibrillation few months back. Abdominal X-rays
demonstrated multiple gas filled dilated small bowel loops with fold thickening.
CT scan abdomen was ordered to exclude:
a) Acute appendicitis
b) Bowel perforation
c) Internal hernia
d) Mesenteric ischemia
e) Small bowel obstruction
Reference: David Sutton Chapter 19, page 630, 7th Edition (Acute abdomen, mesenteric ischemia) (d)
15. A 49 year old gentleman presented to the emergency department with acute pancreatitis. He is a
known alcoholic for 20 years. His initial CT scan showed inflammation of the pancreas. Follow up CT
scan after two weeks showed a well demarcated fluid collection within the pancreas with a bleb of
gas.
The most likely diagnosis is:
a. Acute pseudocyst
b. Enteric fistula
c. Gland necrosis
d. Pancreatic abscess
e. Pancreatic phlegmon
Reference: 7th edition Sutton p 796
16. A middle age lady presented with anemia and ongoing malena for six months. Colonoscopy and blood
biochemistry was normal. Enhanced CT scan abdomen showed a well defined intraluminal hypodense
mass in the jejunum.
The most likely diagnosis is :
a. Adenocarcinoma
b. Carcinoid
c. Leiomyoma
d. Lymphoma
e. Tuberculosis
Reference: 7th edition Sutton p 624-25
17. A 31 years old female 13 weeks pregnant presents with pain and swelling in her left submandibular
region for a few weeks. She is afebrile. The pain increases after taking food. Examination shows
slight tender swelling in left submandibular region. ENT physician wants to exclude submandibular
duct calculus.
The most appropriate initial investigation will be:
a) CT sialography with lead shielding
b) MR sialography without contrast
c) Plain X-ray with lead shielding
d) Sialography with lead shielding
e) Ultrasound of left submandibular region.
Reference: David Sutton Chapter 18, page 537, 7th Edition (salivary gland calculus) (e)
18. An 11 year old girl presents with dysphagia for three weeks. She also has distinct skin lesion on her
hands and feet with blistering. Her 2 other siblings have similar skin lesions. Barium swallow
demonstrated multiple esophageal ulcers and stricture at the distal esophagus. An esophageal web in
upper esophagus was also identified.
The most likely diagnosis is:
a) Crohn’s disease.
b) Epidermolysis bullosa
c) Pemphigoid esophagitis
d) Plummer vinson syndrome
e) Scleroderma
Reference: David Sutton Chapter 18, page 559, 7th Edition (causes and association of esophageal strictures) (b)
19. A 37 years male presented with acute epigastric pain and hematemesis for last two days. He does not
have any known comorbid; however he used to take NSAID s for his migraine quite frequently. His
physician ordered for barium meal examination which revealed multiple rounded ulcers associated
with rim of surrounding halo in antral region.
The most likely diagnosis is:
a) Acute erosive gastritis
b) Crohn’s gastritis
c) H. pylori associated gastritis
d) Pan gastritis
e) Reactive gastritis
Reference: David Sutton Chapter 19, page 579, 7th Edition (causes and appearance of gastritis) (a)
20. A 26 years old female presented with diarrhea and melena for two months. She also complains of low
grade fever and malaise. CBC shows HB of 8g/dl and WBC of 9000. Mauntoux test was negative.
She underwent small bowel follow through which revealed patchy asymmetric ulceration in distal ileum
with separation of bowel close to terminal ileum. Ultrasound showed multiple mesenteric sub
centimeter lymph nodes with small bowel wall thickening:
Most likely possibility is:
a) Tuberculosis
b) Celiac disease
c) Crohn’s disease.
d) Lymphoma
e) Scleroderma
Reference: David Sutton Chapter 19, page 579, 7th Edition (inflammatory bowel disease) (c)
21. A 29 years man presents with a 6 month history of dysphagia associated with retrosternal pain. His
barium swallow demonstrates markedly dilated esophagus containing food debris. There is smooth
narrowing of distal esophagus with barium intermittently spurting into the stomach.
The most likely diagnosis would be:
a. Esophageal achalasia
b. Esophageal leiomyoma
c. Para esophageal hiatus hernia
d. Peptic esophageal stricture
e. Squamous cell carcinoma of esophagus
ANSWER: A
Reference: Grainger, (Chapter 18-20)
22. A 68 years man presented to his GP with 1-month history of epigastric pain, vomiting and mild weight
loss. His examination was unremarkable. His upper GI endoscopy demonstrated mild gastritis.
Biopsies were taken for H. Pylori and he was started on eradication therapy. Three months later his
symptoms persisted and he lost 5 kg weight. A double contrast barium meal was performed which
revealed a shallow ulcer on lesser curvature of stomach.
The characteristic finding differentiating benign ulcer from than malignant ulcer is:
a. Hampton's line is present
b. Nodular mucosal folds stop at the edge of the lesion
c. Ulcer does not extend beyond the gastric wall
d. Ulcer has irregular margins
e. Ulcer measures 40 mm in size
ANSWER: A
Reference: Grainger, (Chapter 18-20)
23. A 49 years man presented to his GP with increasing dysphagia and weight loss. Upper GI endoscopy
revealed a tumor in the distal esophagus and biopsies confirmed esophageal adenocarcinoma. The
patient underwent contrast enhanced CT of the chest and abdomen which showed mucosal thickening
in distal esophagus but no other abnormality. An endoscopic ultrasound was performed which showed
tumor infiltration through the muscularis propria and adventitia but does not extend beyond the serosa.
A round 13 mm peritumoral node is noted.
From this information, the TNM staging of this tumor would be:
a) T2 N0 M0
b) T2 N1 M0
c) T3 N0 M0
d) T3 N1 M0
e) T4 N1 M0
ANSWER: D
Reference: Grainger, (Chapter 18-20)
24. A 52 years man is investigated for weight loss and dysphagia. His endoscopy showed
adenocarcinoma of posterior wall of gastric body which was confirmed on histopathology. A contrast
enhanced CT of abdomen showed focal gastric mucosal thickening with small amount of free fluid in
left paracolic gutter and pelvis. An endoscopic ultrasound was performed which showed tumor
extension beyond the serosal surface of posterior gastric wall.
Structure which is at most risk for direct invasion by this tumor:
a. Abdominal aorta
b. Left lobe of liver
c. Pancreas
d. Right diaphragmatic crus
e. Transverse colon
ANSWER: C
Reference: Grainger, (Chapter 18-20)
25. A 32 years man presented to his GP with increasing pain on swallowing solids and liquids. He has lost
15 kg in weight over the preceding 2 months. After full history and examination, he was found to be
HIV positive with very low CD4 count. His GP referred him for barium swallow examination which
demonstrated a single ulcer in mid-esophagus. This ulcer had smooth margins, measuring 4 cm in
length and is oval in shape. No stricture identified.
Most likely diagnosis would be:
a. Candida oesophagitis
b. CMV oesophagitis
c. Intramural pseudodiverticulosis
d. Esophageal lymphoma
e. Squamous cell carcinoma of esophagus
ANSWER: B
Reference: Grainger, (Chapter 18-20)
26. A 45 years female presented with reflux underwent barium swallow. There was an incidental finding in
cervical esophagus near the cricopharyngeus where a thin membrane of uniform thickness (2mm)
extending from anterior oesophageal wall was visible. Her past medical history included treated
hypothyroidism and investigations for iron deficiency anemia.
Most likely diagnosis would be:
a. Complications of hypothyroidism
b. Epidermolysis
c. Esophageal stricture
d. Plummer-Vinson syndrome
e. Schatzki ring
ANSWER: D
Reference: Grainger, (Chapter 18-20)
27. 32 years female was investigated for severe odynophagia with barium swallow which reproduced
patient's symptoms. Findings were recorded as "compartmentalization of esophagus with numerous
tertiary contractions". Very high pressures were noted on manometeric studies.
The most likely diagnosis would be:
a. Achlasia
b. Chlasia
c. Diffuse oesophageal spasm
d. Plummer-Vinson syndrome
e. Presbyoesophagus
ANSWER: C
Reference: Grainger, (Chapter 18-20)
28. A patient presented with dysphagia initially to liquids then also gradually to solids. His barium swallow
demonstrated a stricture of the distal esophagus and subsequent biopsy confirmed malignancy.
Most likely cell type would be:
a. Adenocarcinoma
b. Leiomyosacoma
c. Lymphoma
d. Metastasis
e. Squamous cell carcinoma
ANSWER: a
Reference: Grainger, (Chapter 18-20)
29. A 62 years woman presented with dysphagia. She was otherwise well but was known to have long
standing diabetes. A barium swallow was performed which showed multiple tiny collections of barium
adjacent to the esophageal lumen. A short distal esophageal stricture was also noted.
The radiological diagnosis would be:
a. Adenomyomatosis
b. Erosive oesophagitis
c. Intramural pseudodiverticulosis
d. Pharyngeal pouch
e. True oesophageal diverticulum
ANSWER: C
Reference: Grainger, (Chapter 18-20)
30. A patient presented to Emergency Department with epigastric and retrosternal pain. A significant
esophageal injury was suspected clinically.
Finding most suggestive of Boerhaave's syndrome rather than Mallory-Weiss tear:
a. Distal esophageal injury
b. Esophageal mucosal irregularity
c. Hematemesis
d. History of alcoholism
e. Pneumomediastinum
ANSWER: E
Reference: Grainger, (Chapter 18-20)
31. A 12 years male child presented with intermittent abdominal pain and vomiting over a two month
period with no weight loss. On examination, there was voluntary guarding in the central abdomen but
no peritonism. His inflammatory markers were raised. His ultrasound showed a trace of fluid but no
structural abnormality. A technetium-99m pertechnetate scan showed uptake in the stomach and in
right iliac fossa on the 20-minute images.
Most likely diagnosis would be:
a. Appendicitis
b. Crohn's disease
c. Meckel's diverticulum
d. Mesenteric adenitis
e. Right ovarian torsion
ANSWER: C
Reference: Grainger, (Chapter 18-20)
32. A patient had recently been diagnosed with gastric carcinoma that had been staged locally as T2
disease.
The extent of tumor penetration through the gastric wall would be:
a. Penetrated through the serosa
b. Invading adjacent organs
c. Limited to the submucosa
d. Limited to the serosa
e. Limited to the mucosa
ANSWER: C
Reference: Grainger, (Chapter 18-20)
GU
1. A 2 year old baby boy was investigated for failure to thrive. His investigations showed decreased
bone density and pseudofractures .His ultrasound kidneys showed nephrocalcinosis .The child
is most likely suffering from :
a) Chronic Glomerulonehritis
b) Fanconi syndrome
c) Nephritic syndrome
d) Renal tubular acidosis
e) Vitamin D-resistant rickets
Reference: 7th edition Dahnert page 153 and 154
2. A 15 year old female with BMI of 18, presents with history of violent attacks of colicky flank pain,
nausea and chills. She has a respiratory rate of 15, pulse of 76/m and B.P of 145/90. Urine DR
reveals hematuria and proteinuria. Patient also suggests a weighing feeling on the abdomen. Pain
is typically relieved by lying down.
U/S and CT abdomen done one month back came back normal.
Disease will be most likely diagnosed on
a) DEXA Scan
b) IVP
c) MAG-3 Scan
d) MRI pelvis
e) Urine CS
3. A 45 year old male presents with dull abdominal pain. He has a history of weight loss,
nausea and vomiting. On examination you observe that the bilateral lower limbs are
swollen along with hydrocele. Vitals are : Temp 39c, B.P 145/90, R.R 16, pulse 89.
Patient has brought an old IVP which shows ureterectasis at L4 level, bilateral medial
deviation of ureter in middle third with gradual tapering of ureter and mild
pyelocaliectasis. You order a CT abdomen which shows periaortic mass of attenuation
similar to muscle.
Most likely diagnosis in this patient will be:
a) Lymphoma
b) Metastasis
c) Renal Cell Carcinoma
d) Retroperitoneal adenopathy
e) Retroperitoneal fibrosis
4. Mother of a 2 year old boy notices an abdominal mass while giving bath, on examination you
notice that the boy has an ataxic gait and limps on walking. You also observe that the boy has
chaotic eye movements. On labs workup there is increased urinary vanillyl mandelic acid (VMA)
and homovanillic acid (HVA) level.
The most likely site of this tumor metastasis is:
a) Bones
b) Liver
c) Lung
d) lymph nodes
e) Orbit
5. A full term baby boy after 3 hours of birth presented with increasing abdominal girth. He had a forceps
delivery after 12 hours of difficult labor. His CBC then performed, shows low hemoglobin level.
The most likely diagnosis is:
a) Adrenal hemorrhage
b) Bilalteral hydronephrosis
c) Bowel hematoma
d) Congenital neuroblastoma
e) Splenic hematoma
Reference: Radiology Review Manual, Wolfgang Dahnert, 6th Edition, page 918
6. A 22 year lady underwent plain CT scan of abdomen for suspicion of ureteric calculus. A single stone
was seen in the left mid ureter. Incidentally a 2 cm nodule was noted in the right adrenal gland. The
Hounsfield unit was 20 HU.
The most appropriate next step in management is:
a) Arterial & Delayed phase CT
b) Conventional Angiography
c) CT guided biopsy of nodule
d) Follow-up CT in three months
e) Ultrasound of abdomen
Reference: www.STATdx.com
Reference: Radiology Review Manual, Wolfgang Dahnert, 6th Edition, page 881
7. A 32 year gentleman presented with pain in right hypochondrium and vomiting for 24 hours. His
physical examination was indeterminate. To exclude appendicitis he underwent plain CT of the
abdomen. No evidence of appendicitis seen, however a mass measuring 5 cm is seen in lower pole of
left kidney containing hyper and hypodense components. The Hounsfield unit of hypodense component
is -80 HU.
The most appropriate next step in management is:
a) Annual follow-up
b) Color Doppler ultrasound
c) Contrast enhanced CT scan
d) Intravenous pyelogram
e) Semiannual follow-up
Reference: Radiology Review Manual, Wolfgang Dahnert, 6th Edition, page 922
8. A 5 year baby boy presented with abnormal abdominal wall with leakage of fluid positive for urine. X-
ray abdomen shows diastases of symphysis pubis. Child also has epispadia.
The most likely diagnosis is:
a) Bladder Exstrophy
b) Omphalocele
c) Patent Urachus
d) Prune Belly Syndrome
e) Vesical Fistula
Reference: Radiology Review Manual, Wolfgang Dahnert, 6th Edition, page 923
9. A 22 year gentleman presented at 4 am in emergency room with acute scrotal pain which woke him up
from sleep. On examination scrotum is tender, erythematous and enlarged. Ultrasound shows increased
vascularity and no evidence of collection.
The most likely diagnosis is:
a) Epididymoorchitis
b) Scrotal abscess
c) Scrotal Hydrocele
d) Testicular Torsion
e) Testicular tumor
Reference: Radiology Review Manual, Wolfgang Dahnert, 6th Edition, page 926
10. A 66 year gentleman presented to clinic with fever and scrotal pain for 2 days. He has history of
diabetes for 15 years. On examination the scrotum is enlarged, tender, and erythematous with
crepitations. His Laboratory work shows elevated WBC.Ultrasound shows thickened scrotal wall and
dirty acoustic shadowing.
The most likely diagnosis is:
a) Epididymoorchitis
b) Fournier gangrene
c) Scrotal Abscess
d) Testicular Torsion
e) Testicular Tumor
Reference: Radiology Review Manual, Wolfgang Dahnert, 6th Edition, page 928
11. Routine ultrasound of abdomen of 30 year lady for insurance purposes shows malrotated kidneys with
fused lower poles in the midline.
The location of fusion of horseshoe kidneys is:
a) L3-L4 anterior to aorta and posterior to inferior mesenteric artery
b) L4-L5 anterior to aorta and posterior to superior mesenteric artery
c) L4-L5 anterior to aorta and posterior to inferior mesenteric artery
d) L3-L4 anterior to aorta and posterior to superior mesenteric artery
e) L5-S1 anterior to aorta and posterior to inferior mesenteric artery
Reference: Radiology Review Manual, Wolfgang Dahnert, 6th Edition, page 929
12. A 70 year lady presented with severe pain in right lumbar area with high grade fever for 48 hours. Her
labs show urinary tract infection. She has history of diabetes for 3 years and recurrent UTIs. She
underwent CT urography examination. The right kidney is enlarged with distorted pelvicaliceal system
and low attenuation mass. Multiple 5-8 mm dome shaped smooth mural filling defects identified in the
right renal pelvis and upper 2/3 of ureter.
The most likely diagnosis is:
a) Malakoplakia
b) Multifocal TCC
c) Pyeloureteritis cystica
d) Urinary Tuberculosis
e) Xanthogranulomatous Pyelonephritis
Reference: Radiology Review Manual, Wolfgang Dahnert, 6th Edition, page 933
13. 55 year old female with history of diabetes and chronic urinary tract infections since 5 years came for a
intravenous urography (IVP). Multiple round smooth lucent filling defects measuring approximately 1-
3 mm in the distal 1/3rd of left ureter and bladder were seen. Previous IVP 2 years back also showed
similar findings without significant change in appearance.
The most likely diagnosis is:
a) Malakoplakia
b) Multifocal TCC
c) Multiple Polyps
d) Pyeloureteritis cystica
e) Renal Tuberculosis
Reference: Radiology Review Manual, Wolfgang Dahnert, 6th Edition, page 953
14. A 19 year young man brought to the emergency by paramedics after road traffic accident. On
examination he had multiple injuries with severe blood loss. After initial resuscitation he underwent CT
scan which showed liver laceration. Both kidneys showed enhancement of medulla and non
enhancement of cortex; however the subcapsular cortical rim was enhancing.
The most likely cause of this appearance of kidneys is;
a) Acute cortical necrosis
b) Acute main renal artery occlusion
c) Acute tubular necrosis
d) Chronic urinary obstruction
e) Renal vein thrombosis
Reference: Radiology Review Manual, Wolfgang Dahnert, 6th Edition, page 916.
15. A 52 year gentleman with history of diabetes underwent contrast CT scan for abdominal pain. The next
day he presented with very low urinary output. Ultrasound examination of kidneys showed bilateral
enlarged smooth kidneys with reduced echogenicity of medulla and normal echogenicity of cortex. Few
hours later his renal output improved and subsequently normalized in the next 24 hours.
The most likely diagnosis is:
a) Acute cortical necrosis
b) Acute glomerulonephritis
c) Acute tubular necrosis
d) Renal artery occlusion
e) Renal vein thrombosis
Reference: Radiology Review Manual, Wolfgang Dahnert, 6th Edition, page 917
16. 20 year female came for growth scan of fetus at 32 weeks of pregnancy. Fetus had bilateral
hydronephrosis. In the right kidney the hydronephrosis had caused the central sinus echoes of ovoid
configuration with continuous echogenic sinus periphery.
The hydronephrosis in the left kidney had caused discontinuity of echogenic sinus periphery.
The grade of hydronephrosis in both kidneys is;
a) Grade 0 in right and grade 1 in left
b) Grade 0 in right and grade 2 in left
c) Grade 1 in right and grade 3 in left
d) Grade 2 in right and grade 4 in left
e) Grade 3 in right and grade 2 in left
Reference: Radiology Review Manual, Wolfgang Dahnert, 6th Edition, page 930
17. A 1 year infant presents with bilateral undescended testis. Patient has normal external male genitalia.
Ultrasound scan confirmed left testis in the inguinal canal however the right testes is not visualized.
What is the next imaging modality of choice for detection of right testis;
a) Computed tomography
b) Laproscopy examination
c) Magnetic resonance Imaging
d) Ultrasound in 1 week,
e) Venography emanination
Reference: Frush DP, Sheldon CA. Diagnostic imaging for pediatric scrotal disorders. Radiographics. 1998
Jul-Aug;18(4):969-85. Review
18. 25 year male presents with scrotal pain. Ultrasound is advised which shows multiple 1-2 mm
hyperechoic nonshadowing foci scattered throughout the parenchyma of left testes. No mass lesion is
identified.
The next step in management of this patient is;
a) No follow up needed
b) 6 monthly follow up
c) 12 monthly follow up
d) 24 monthly follow up
e) 5 yearly follow up
Reference: Radiology Review Manual, Wolfgang Dahnert, 6th Edition, page 974
19. 17 year male presented to emergency with scrotal pain. Ultrasound is advised which shows multiple
scattered 1-2 mm hyperechoic nonshadowing foci scattered throughout the parenchyma of left testes.
No mass lesion is identified.
The most common association of this finding is;
a) Cryptochordism
b) Down’s Syndrome
c) Infertility
d) Klinefelter syndrome
e) Germ cell tumor
Reference: Radiology Review Manual, Wolfgang Dahnert, 6th Edition, page 974
20. 44 year male presents with low grade fever, weight loss, and abdominal pain. CT scan is advised which
shows multiple non-necrotic lymph nodes in retroperitoneum. Multiple homogenous poorly marginated
nodular masses hypodense to renal parenchyma and showing less enhancement compared to kidneys.
The most likely diagnosis is;
a) Multifocal renal cell carcinoma
b) Multiple renal abscesses
c) Renal Lymphoma
d) Renal metastases
e) Xanthogranulomatous pyelonephritis
Reference: Radiology Review Manual, Wolfgang Dahnert, 6th Edition, page 932
21. A 26 year female was advised intravenous urography for renal colic. Thick dense streaks of contrast
material were seen radiating from pyramids peripherally. Rest of the examination is normal. Ultrasound
was also performed which shows echogenic medulla bilaterally.
The most likely diagnosis is;
a) Acute pyleonephritis
b) Autosomal recessive PCKD
c) Medullary cystic disease
d) Medullary sponge Kidney
e) Normal variant
Reference: Radiology Review Manual, Wolfgang Dahnert, 6th Edition, page 935
22. 3 month baby boy is sent for ultrasound for evaluation of increasing abdominal girth. On examination
mass was felt by primary physician. On ultrasound examination the right kidney is enlarged however
otherwise appears normal. The left kidney is significantly enlarged with multiple cysts randomly spread
throughout left kidney in varying shapes and sizes separated by septa. The normal renal architecture
and central sinus complex is not visualized.
The most likely diagnosis is;
a) Gross Hydronephrosis
b) Meckel-Gruber Syndrome
c) Multicystic Dysplastic Kidney
d) Multilocular cystic renal tumor
e) Polycystic kidney disease
Reference: Radiology Review Manual, Wolfgang Dahnert, 6th Edition, page 937
23. 10 day old baby presented with abdominal mass. Ultrasound shows left kidney moderate
hydronephrosis.
The most common cause of congenital hydronephrosis,
a) Ectopic Ureterocele
b) Posterior urethral Valves
c) Prune Belly Syndrome
d) Ureteropelvic junction obstruction
e) Vesicoureteral Reflux
Reference: Radiology Review Manual, Wolfgang Dahnert, 6th Edition, page 931
24. 2 month baby girl presented with abdominal mass. Ultrasound of right kidney shows a single
echogenic mass with concentric rings of alternating higher echogenicity. No acoustic shadowing to
suggest calcifications, no evidence of venous or renal pelvis invasion.
Based on these findings the most likely diagnosis is
a) Mesoblastic nephroma
b) Nephroblastomatosis
c) Renal cell carcinoma
d) Rhabdoid tumor of kidney
e) Wilms Tumor
Reference: Radiology Review Manual, Wolfgang Dahnert, 6th Edition, page 936
25. 2 month baby girl presented with abdominal mass. Ultrasound of right kidney shows right renal mass.
The most common renal neoplasm in this age group is,
a) Mesoblastic nephroma
b) Nephroblastomatosis
c) Renal cell carcinoma
d) Rhabdoid tumor of kidney
e) Wilms Tumor
Reference: Radiology Review Manual, Wolfgang Dahnert, 6th Edition, page 936
26. 2 year boy presented with weight loss and abdominal distension. On examination abdominal mass can
be palpated. Enhanced CT scan is performed which shows large suprarenal mass with stippled
calcification displacing the kidney inferiorly and retroperitoneal lymph nodes. Mass is also crossing the
midline encasing the IVC and aorta. No distant metastasis is seen.
The stage of the disease is;
a) Stage I
b) Stage II
c) Stage III
d) Stage IV
e) Stage V
Reference: Radiology Review Manual, Wolfgang Dahnert, 6th Edition, page 941
27. 3 year girl presented with weight loss and abdominal distension. Enhanced CT scan is performed which
shows large heterogeneous suprarenal mass with stippled calcification displacing the kidney inferiorly.
Mass is also crossing the midline encasing the IVC and aorta.
The most likely diagnosis is;
a) Adrenocortical carcinoma
b) Neuroblastoma
c) Pheochromocytoma
d) Retroperitoneal teratoma
e) Wilms Tumor
Reference: Radiology Review Manual, Wolfgang Dahnert, 6th Edition, page 941
28. 5 year old boy comes with abdominal pain and distension. On examination abdominal mass can be
palpated. Patient also has hypertension. Enhance CT scan shows bilateral renal masses. The right renal
mass is showing invasion into IVC.
The most likely diagnosis is;
a) Mesoblastic nephroma
b) Nephroblastomatosis
c) Neuroblastoma
d) Renal cell carcinoma
e) Wilms Tumor
Reference: Radiology Review Manual, Wolfgang Dahnert, 6th Edition, page 993
29. IVP examination of 20 year female with history of intake of analgesics due to sickle cell disease shows
normal sized right kidney. However the calyceal system is abnormal with appearance of fornix
widening, ring and claw configuration, filling defects and deformed club shape.
The most likely diagnosis is;
a) Congenital megacalices
b) Hydronephrosis
c) Necrotizing papilltis
d) Postobstructive renal atrophy
e) Pyelonephritis
Reference: Radiology Review Manual, Wolfgang Dahnert, 6th Edition, page 943
30. A 17 year man is diagnosed with bilateral suprarenal masses. 2 years back he also had neck surgery
after a mass was discovered that was elevating calcium.
What else is he at risk for;
a) Adrenal adenocarcinoma
b) Hemangioblastoma
c) Medullary carcinoma thyroid
d) Osteoma
e) Pancreatic tumor
Reference: Radiology Review Manual, Wolfgang Dahnert, 6th Edition, page 944
31. 45 year gentleman presents with repeated episodes of sweating, flushing and palpitations several times
a day. Urine examination shows elevated epinephrine. Enhanced abdominal CT shows a complex mass
8 x 8 cm compressing the upper pole of left kidney.
The most likely diagnosis is;
a) Adrenal adenoma
b) Adrenal hemorrhage
c) Adrenal Metastasis
d) Adrenal Pheochromocytoma
e) Adrenocortical carcinoma
Reference: Radiology Review Manual, Wolfgang Dahnert, 6th Edition, page 944
32. A 42 year lady is referred for contrast CT due to abdominal pain and sweating. 15 minutes after
contrast injection she starts to complain of headaches, palpitation and starts sweating. After initial
reassurance blood pressure obtained was 210/105.
Administration of which of the following could have prevented such an attack;
a) Alpha blocker
b) Anti-histamine
c) Beta blocker
d) Sedatives
e) Steroids
Reference: Radiology Review Manual, Wolfgang Dahnert, 6th Edition, page 944
33. A 42 year lady is referred for contrast CT due to abdominal pain, episodes of flushing and sweating. 15
minutes after contrast injection she starts to complain of headaches, palpitation and starts sweating.
After initial reassurance, blood pressure was 210/105. What is the cause of this complication
a) Allergic reaction
b) Anxiety attack
c) Claustrophobia
d) Hypertensive crisis
e) Myocardial infarction
Reference: Radiology Review Manual, Wolfgang Dahnert, 6th Edition, page 944
34. A 23 years young man is brought to the emergency with unconsciousness. A witness said that he
complained of sudden onset of severe headaches before fainting. Ultrasound of the abdomen showed
bilateral enlarged cystic kidneys.
The most likely diagnosis is;
a) Brain Stroke
b) Brain tumor
c) Malingering
d) Meningitis
e) Subarachnoid hemorrhage
Reference: Radiology Review Manual, Wolfgang Dahnert, 6th Edition, page 945
35. A 48 years young man comes for ultrasound due to vague abdominal pain since 6 months. On
examination his physician felt bilateral abdominal masses. Ultrasound shows enlarged kidneys with
multiple cortical cysts distorting the renal outline. Creatinine and BUN levels are normal. Few simple
cysts are also seen in liver.
What additional imaging should be performed;
a) Barium Swallow
b) Chest X-ray
c) CT Brain Angiography
d) Cystogram
e) Plain MRI Brain
Reference: Radiology Review Manual, Wolfgang Dahnert, 6th Edition, page 945
36. 2 day old newborn baby boy is brought with complain of urine dribbling. VCUG is performed which
shows trabeculated urinary bladder. Grade II reflux seen bilaterally. During voiding prostatic urethra
became dilated and minimal streak of urine was seen passing in penile urethra.
The most likely diagnosis is;
a) Primary megaureter
b) Posterior urethral valves
c) Urethral stenosis/stricture
d) Ureterovesical obstrcution
e) Vesicoureteral reflux
Reference: Radiology Review Manual, Wolfgang Dahnert, 6th Edition, page 947
37. 72 year old gentlemen presents with weight loss. Blood work revealed significantly elevated PSA.
Enhanced MRI is performed which shows a hypointense lesion measuring 1.9 cm in right posterior
peripheral zone on T2. The right seminal vesicle also appears low signal on T2 sequences.
The stage of prostatic cancer is;
a) T1
b) T2a
c) T2b
d) T3b
e) T3c
Reference: Radiology Review Manual, Wolfgang Dahnert, 6th Edition, page 948
38. 66 year gentlemen complain of difficulty passing urine. Laboratory examination shows normal white
cell count and elevated PSA. TRUS is advised which shows asymmetric enlargement of prostate with
hypoechoic lesion measuring 0.8 x 0.5 cm in the left peripheral zone causing bulge in capsule.
The most likely diagnosis is;
a) Benign hyperplasia
b) Prostatic abscess
c) Prostatic cancer
d) Prostatic cyst
e) Sonographic artifact
Reference: Radiology Review Manual, Wolfgang Dahnert, 6th Edition, page 949
39. A 20 day newborn presents for ultrasound examination as prenatal ultrasound showed bilateral
hydronephrosis. On examination he also has a wrinkled lower anterior abdominal wall and his testes are
not felt in scrotum. Ultrasound shows massively dilated tortuous elongated ureters with poor peristalsis.
40. Ultrasound examination of a 20 day newborn is advised as prenatal ultrasound showed bilateral
hydronephrosis. During ultrasound examination you notice his lower anterior abdominal wall is
wrinkled. Ultrasound shows massively dilated tortuous elongated ureters with poor peristalsis.
He is also likely to have;
a) Cryptochordism
b) Hirschsprung Disease
c) Malrotation
d) Pulmonary hypoplasia
e) Scoliosis
Reference: Radiology Review Manual, Wolfgang Dahnert, 6th Edition, page 949
41. A 22 year man comes for intravenous urography due to right renal colic. The right renal outline was
enlarged with evidence of two complete pelvicalyceal system. No kidney was seen in left renal region.
Ultrasound was also performed to confirm the presence of crossed fused ectopia.
The two ureters of crossed fused kidneys insert in bladder in following location;
a) Upper moiety drain inserts in ectopic location in ipsilateral side of bladder and lower moiety
inserts orthotopically in contralateral side of bladder
b) Upper moiety drain inserts orthotopically in contralateral side of bladder and lower moiety
inserts in ectopic location in contralateral side of bladder
c) Upper moiety drain inserts orthotopically in ipsilateral side of bladder and lower moiety inserts
in ectopic location in contralateral side of bladder
d) Upper moiety drain inserts orthotopically in ipsilateral side of bladder and lower moiety inserts
in ectopic location in ipsilateral side of bladder
e) Upper moiety drain inserts orthotopically in ipsilateral side of bladder and lower moiety
inserts orthotopically in contralateral side of bladder
Reference: A text book of radiology and imaging: volume 2, David Sutton, 7th edition, page 932
42. 60 year male refereed for ultrasound of testes due to increased size of scrotum. Hypoechoic mass
measuring 1.5 X 1.2 cm with invasion of epididymis. Mild hydrocele is seen.
The TNM stage of the disease is;
a) Tis
b) T1
c) T2
d) T3
e) T4
Reference: Radiology Review Manual, Wolfgang Dahnert, 6th Edition, page 980
43. 55 year old female presents with complain of weight loss and blood in urine. Ultrasound of bladder
shows a nonmobile polypoidal hypoechoic mass protruding into lumen measuring 1.5 cm and showing
vascularity. No acoustic shadowing is seen.
The most likely diagnosis is;
a) Focal Cystitis
b) Hemorrhagic Clot
c) Metastasis
d) Primary malignancy
e) Vesical calculus
Reference: Radiology Review Manual, Wolfgang Dahnert, 6th Edition, page 980
44. 62 year male is referred for MRI pelvis for staging of bladder tumor seen on ultrasound. On MRI
examination a 2.2 cm polypoidal mass is seen. On T1 sequences the adjacent perivesical fat is dirty.
The appropriate stage of the disease is;
a) T1
b) T2a
c) T2b
d) T3a
e) T3b
Reference: Radiology Review Manual, Wolfgang Dahnert, 6th Edition, page 980
45. 26 year male come with fullness in scrotum. On examination the left testes appeared bulky to primary
physician who referred him for scrotal ultrasound. Ultrasound examination a focal uniformly
hypoechoic slightly lobulated mass measuring 1.5 x 1 cm confined within tunica albuginea. Beta hcg is
elevated
The most likely diagnosis is;
a) Leukemia
b) Mestasatsis
c) Seminoma
d) Teratoma
e) Testicular cyst
Reference: Radiology Review Manual, Wolfgang Dahnert, 6th Edition, page 976
46. 22 year old comes is referred for ultrasound testes. patient complains of vague heaviness feeling in
testes. On ultrasound examination a well defined mass is identified in right testes measuring
approximately 2 cm with alternating hypoechoic and hyperechoic rings. Subsequently MRI was
performed which shows low signal intensity fibrous capsule with central high signal on T2 and T2
giving a target appearance.
The most likely diagnosis is;
a) Choriocracinoma
b) Epidermoid cyst
c) Malignant Teratoma
d) Seminoma
e) Sertoild cell tumor
Reference: Radiology Review Manual, Wolfgang Dahnert, 6th Edition, page 976
47. 2 year old came with history of undescended testis. On examination both testis are now seen in
scrotum.
Patient is at most risk of;
a) Choriocarcinoma
b) Embryonal cell carcinoma
c) Leydig cell tumor
d) Seminoma
e) Teratoma
Reference: Radiology Review Manual, Wolfgang Dahnert, 6th Edition, page 976
48. 45 year old female presents with fever and draining sinus in the left loin. She has history of chronic left
side renal infections since 6 years. CT scan is performed which shows a heterogenous enlarged left
kidney with large staghorn calculus. Perinephric collection is seen involving the left psoas muscle and
abdominal wall with the formation of a cutaneous fistula.
49. 19 year male comes for CT abdomen for evaluation of renal colic. CT does not show any urolithiasis
however a 3.5 cm cyst is seen in right kidney with well defined wall showing nodular calcifications. Its
hounsfiled unit is 90HU.
The Bosniak Classification of this cyst is;
a) Class1
b) Class 2
c) Class 2F
d) Class 3
e) Class 4
Reference: A text book of radiology and imaging: volume 2, David Sutton, 7th edition, page 944
Reference: Radiology Review Manual, Wolfgang Dahnert, 6th Edition, page 962
50. 63 year male is referred for CT abdomen for evaluation of weight loss and abdominal pain. A 4cm
cortical cyst is visualized in right kidney at the lower pole with irregular thickened septa showing slight
enhancement, irregular and thickened calcifications, irregular margination and uniform wall thickening.
The Bosniak Classification of this cyst is;
a) Class 1
b) Class 2
c) Class 2f
d) Class 3
e) Class 4
Reference: A text book of radiology and imaging: volume 2, David Sutton, 7th edition, page 944
Reference: Radiology Review Manual, Wolfgang Dahnert, 6th Edition, page 962
51. 71 year male is referred for ultrasound for evaluation of hematuria and weight loss. A 3cm cyst is
visualized in interpolar region of left kidney showing irregular thickened septa with thick hyperechoic
calcification with posterior acoustic shadowing. The cyst wall shows irregular with nodular masses.
The Bosniak Classification of this cyst is;
a) Class 1
b) Class 2
c) Class 2f
d) Class 3
e) Class 4
Reference: A text book of radiology and imaging: volume 2, David Sutton, 7th edition, page 944
Reference: Radiology Review Manual, Wolfgang Dahnert, 6th Edition, page 962
52. 55 year female is referred for ultrasound for annual health check. A 2.5cm cyst is visualized in upper
pole of right kidney showing irregular thickened septa with thick hyperechoic calcification with
posterior acoustic shadowing. The cyst wall shows uniform thickness.
The next step in management;
a) 3 month follow
b) Aspiration of cyst
c) No follow needed
d) Surgical excision
e) Yearly follow up
Reference: A text book of radiology and imaging: volume 2, David Sutton, 7th edition, page 944
Reference: Radiology Review Manual, Wolfgang Dahnert, 6th Edition, page 962
53. 24 year female comes for CT abdomen for evaluation of renal colic. CT does not show any urolithiasis
however a 3.1 cm cyst is seen in left with well defined wall and thin septum. Its hounsfiled unit is
40HU.
The next step in management;
a) 3 month follow
b) Aspiration of cyst
c) No follow needed
d) Surgical excision
e) Yearly follow up
Reference: A text book of radiology and imaging: volume 2, David Sutton, 7th edition, page 944
Reference: Radiology Review Manual, Wolfgang Dahnert, 6th Edition, page 962
54. 38 year male with history of chronic kidney diseases since many years now presents with new onset
hypertension 6 months after renal transplant surgery. Ultrasound doppler of ttransplant renal artery is
perfomred. It shows peak systolic velocity greater than 200 cm/sec away from kidney and spectral
broadening with tardus-parvus waveform close to kidney.
Reference: Radiology Review Manual, Wolfgang Dahnert, 6th Edition, page 956
55. 24 hours after renal transplant patient is refered for urgent ultrasound to evaluate cause of abrupt onset
of oliguria. Ultrasound shows enlarged hypoechoic transplant kidney with loss of corticomedullary
differentiation. Arterial doppler shows U-shaped reversal of distolic arterial flow.
The most likely diagnosis is;
a) Acute translant rejection
b) Acute tubular necrosis
c) Renal artery stenosis
d) Renal vein thrombosis
e) Ureteral obstruction
Reference: Radiology Review Manual, Wolfgang Dahnert, 6th Edition, page 969
56. 6 hours after renal transplant patient is brought to the radiology department for evaluation of abrupt
onset of oliguria. Ultrasound shows slight increase in size. Otherwise appears unremarkable. Tc99m
DTPA renal scintigram is performed next which shows complete absence of renal flow and excretion.
The most likely diagnosis is;
a) Acute translant rejection
b) Acute tubular necrosis
c) Hyperacute rejection
d) Renal artery stenosis
e) Renal vein thrombosis
Reference: Radiology Review Manual, Wolfgang Dahnert, 6th Edition, page 963
57. 62 year female presents with weight loss and hematuria. Contrast CT abdomen shows an enhancing
mass in the right upper pole measuring 8 cm. The mass appears to be extending into the renal vein. Post
nephrectomy mass was confirmed as renal cell carcinoma.
The TNM stage of the malignancy is ;
a) T2
b) T3a
c) T3b
d) T4a
e) T4b
Reference: Radiology Review Manual, Wolfgang Dahnert, 6th Edition, page 958
58. 28 year man was hit by a car as he was crossing a street. In the emergency he complained of left loin
pain. His urine was reddish in color. Contrast CT scan is advised which shows a 2 cm laceration at the
lower pole involving the cortex and medulla.
The injury can be classified as;
a) Grade 1
b) Grade 2
c) Grade 3
d) Grade 4
e) Grade 5
Reference: Radiology Review Manual, Wolfgang Dahnert, 6th Edition, page 981
59. 19 year man is brought to the emergency ofter a gang beat him with a cricket bat. He had multiple
bruises the worst over the left renal area. Ultrasound FAST is positive. CT scan was performed which
shows a large perirenal hematoma. The tail of pancreas also shows parecnchymal laceration with duct
injury.
Reference: Radiology Review Manual, Wolfgang Dahnert, 6th Edition, page 981&809
60. Ultrasound of a 40 year female is performed for evaluation of dull pain in abdomen. Bilateral mild
hydronephrosis is seen but ureters are obscured by bowel gases. IVP examination is performed for
urolithiasis. Redemonstration of mild bilateral hydronephrosis. Both ureters are medially deviated with
smooth tapering in the mid portion.
61. IVP examination of a 26 year female with history of recurrent cytitis and incomplete voiding infertility
is performed due to complain of pain radiating to right groin. Mild right ureter hydronephrosis is seen
however no evidence of calculus. In addition the right pelvic ureter is displaced medially.
a) Abdominopelvic lipomatosis
b) Aneurysmal dilatation of iliac vessels
c) Bladder diverticulum at UVJ
d) Hypertrophy of iliopsoas muscle
e) Multiple uterine fibroids
Reference: Radiology Review Manual, Wolfgang Dahnert, 6th Edition, page 896
62. 25 year old female is referred for IVP examination to evaluate left hydronephrosis seen in ultrasound
examination. She also has history of repeated UTI , infertility and reddish urine that starts every month.
IVP shows a 2 cm abrupt smooth stricture in mid left ureter with proximal mild hydroureter.
a) Amyloidosis
b) Endometriosis
c) Schistosomiasis
d) Traumatic
e) Tuberculosis
Reference: Radiology Review Manual, Wolfgang Dahnert, 6th Edition, page 896
63. 60 male comes with complain of inability to micturate. Patient also has fever and perineal tenderness.
Initial attempt to cathetrize failed as patient has significant pain. CT is performed which shows an
enlarged prostate with 1 cm hypodense focus in right side of prostate. The center of the cyst shows a
housfild of -1000 HU. Adjacent periprostatic fat stranding is seen.
a) Metastases
b) Prostatic abscess
c) Prostatic cancer
d) Prostatic cyst
e) Prostatitis
64. A 42 years old gentleman, a known case of testicular microlithiasis for last 5 years, came for follow up
ultrasound. He was found to have a 2 X 2 cm mass in the right testis and paraaortic lymphadenopathy.
The most characteristic feature of the testicular mass is:
a. Bladder diverticulum
b. Cowper’s gland
c. Patent processus vaginalis
d. Prostatic utricle
e. Seminal vesicle cyst
Reference: David Sutton 7th edition page 1017
68. Acute renal failure within 2 days of contrast media injections is defined as an increase of serum creatinine more
than:
a) 1 mg/dL
b) 2 mg/dL
c) 3mg/dL
d) 4 mg/dL
e) 5 mg/dL
69. 50 year old diabetic gentleman underwent contrast enhance CT scan for paranephric collection. He developed
life threatening lactic acidosis.
The most likely cause of lactic acidosis in this patient is:
Sutton Ch 33 pg 1041
3. 32 year female p2+1 and g 3 presented in your ultrasound clinic for a scheduled anomaly scan at 22 weeks of
gestation. Previous dating scan showed viable fetus of 8 weeks. You perform growth scan as part of the
protocol.
Accurate method of assessing Biparietal diameter on a standard axial plane passing through the widest
portion of skull is:
a) Cavum septum pellucidum and interhemispheric fissure in midline
b) Cavum septum pellucidum and third ventricle in midline
c) Thalami and cavum septum pellucidum in midline
d) Thalami and interhemispheric fissure in midline
e) Thalami and third ventricle in midline
Ans: Thalami and cavum septum pellucidum in midline
Sutton Ch 33 page 1042
4. A 22-year woman, primigravida, came for dating scan. LMP corresponding to 7 weeks of gestation. No active
complains. Her transabdominal ultrasound showed gestational sac of 40mm with no fetal pole or cardiac
flicker.
A living embryo should be seen once the gestational sac diameter exceeds size of:
a) 12 to 20mm
b) 17 to 30mm
c) 22 to 35mm
d) 25 to 35mm
e) 9 to 18 mm
Ans: 17 to 30mm
Sutton Ch 33 page 1043
5. A 40-year-lady, P2+2 and G5, with history of 2 abortions in past 3 years. She has raised maternal serum alpha-
fetoprotein (MSAFP) and has come for anomaly scan. LMP corresponding to 20 weeks of gestation. No active
complains. Her transabdominal ultrasound showed neural tube defect.
Commonest neural tube defect is:
a) Acrania
b) Anencephaly
c) Encepahalocele
d) Myelocele
e) Myelomeningocele
Ans: Anencephaly
Sutton Ch 33 page 1050
6. 45 year lady came to ultrasound suite for anomaly scan. She is a high risk patient with mitral valve stenosis and
has precious pregnancy after 10 years of marriage. Sonologist makes a diagnosis of congenital diaphragmatic
hernia.
Sonographic diagnosis of fetal CDH relies on:
a) absence of a normally positioned stomach
b) mediastinal displacement and lung hypoplasia
c) polyhydramnios and pleural effusion
d) Small abdominal circumference.
e) visualization of abdominal organs in the chest
Ans: visualization of abdominal organs in the chest
7. 22 primigravida came for routine anomaly scan. Her anomaly scan revealed a complex echogenic mass in fetal
thorax poterolaterally displacing heart anteriorly. No abnormal vascularity on doppler study. Normal abdominal
circumfrence.
The most likely diagnosis is:
a) Congenital diaphragmatic hernia
b) Cystic adenomatoid malformation.
c) Cystic Hygroma
d) Fetal hydrops
e) Pulmonary sequestration
Ans:CCAM
Sutton Ch 33 page 1057
9. A 25 year old lady presented with dysmenorrhea and dyspareunia for last one year. She had fixed pelvic organs
on bi-manual pelvic examination.
The most commonly affected organ by this condition is:
a. Fallopian tubes
b. Ovaries
c. Recto sigmoid
d. Uterine serosal surface
e. Uterosacral ligament
Reference: 6th edition Dahnert page 1040
10. A 40 year old multigravida presented with painless vaginal bleeding at 36 week of gestation.
The most likely cause of her bleeding is:
a. Placenta accrete
b. Placenta increta
c. Placenta percreta
d. Placenta previa
e. Placental abruption
Reference: 6th edition Dahnert page 1060
11. A 59 year old lady underwent enhanced CT scan of abdomen and pelvis for staging of ovarian cancer. CT
showed disease limited to pelvis; however the ovarian mass was inseparable from uterus. Clear fat planes
between the mass bladder and rectum seen.
The stage of tumor according to FIGO system is:
a. Ib
b. Ic
c. II a
d. II b
e. III
th
Reference: 6 edition Dahnert page 1053
12. A primigravida with amenorrhea of 7 weeks, presented with persistent lower abdomen pain and PV bleeding.
Her pelvic ultrasound showed gestational sac with fetal pole with no cardiac activity and her Os was dilated.
The most likely diagnosis is:
a. Complete abortion
b. Incomplete abortion
c. Inevitable abortion
d. Missed abortion
e. Threatened abortion
Reference: 6th edition Dahnert page 1027